KN³kmμkarniBnænigeroberog elak

lwm plÁún nig elak Esn Bisidæ KN³kmμkarRtYtBinitü

elak elak elak elak

lwm qun Titü em:g RBwm sunitü elak nn; suxNa

elak Gwug sMNag GñkRsI Tuy rINa elak pl b‘unqay

GñkRtYtBinitüGkçraviruTæ elak lwm miKÁsir

karIkMuBüÚTr½

GñkrcnaRkb

kBaØa lI KuNÑaka

elak lwm plÁún

-i-

GarmÖfa esovePA sikSaKNitviTüaedayxøÜnÉg Epñk cMnYnkMupøic EdlGñksikSakMBugkan;sikSaenAkñúgédenH eyIgxJMú)anxitxMRsavRCavcgRkgeLIg kñúgeKalbMNgTukCaÉksa sikSabEnßmelIemeroncMnYnkMupøicedayxøÜnÉg . enAkñúgesovePAenHrYmmanbICMBUkKW CMBkU TI1 emeronsegçbP©ab;CamYy]TahrN_KMrU CMBUkTI2 lMhat;eRCIserIsnigdMeNaHRsay nig CMBUkTI3 CalMhat;Gnuvtþn_ . esovePAenHminl¥hYseK hYsÉgenaHeT kMhusedayGectnaR)akdCaman GaRs½yehtuenH eyIgxJMúCaGñkniBnæ nig eroberog rgcaMCanic©nUvmtiriHKn;BIsMNak; GñksikSakñúgRKb;mCÄdæanedaykþIrIkray edIm,IEklMGesovePAenH[kan;Etman suRkitüPaBbEnßmeTot . CaTIbBa©b;eyIgxJMúsUmCUnBrcMeBaHGñksikSaTaMgGs;CYbEtsuPmgÁl suxPaBl¥ nigTTYlC½yCMn³kñúgkarsikSa nig muxrbrkargar RKb;eBlevla .

át´dMbgéf¶TI 28 mIna 2011

GñkniBnæ lwm plþún Tel : 017 768 246 www.mathtoday.wordpres.com - ii -

lwm plþún nig Esn Bisidæ

rkßasiT§iRKb´y¨ag - iii -

matikarerOg CMBUkTI1

1-niymn½y 2-RbmaNviFIelIcMnYnkMupøic 3-cMnYnkMupøicqøas; 4-Gnuvtþn_cMnYnkMupøickñúgdMeNaHRsaysmIkardWeRkTIBIr 5-cMnYnkMupøickñúgbøg; 6-m:UDúléncMnYnkMupøic 7-GaKuym:g;éncMnYnkMupøic 8-TRmg;RtIekaNmaRténcMnYnkMupøic 9-RbmaNviFIelIcMnYnkMupøictamTRmg;RtIekaNmaRt 10-bMElgvilCMuvijKl;tRmuyénbøg;kMupøic 11-b¤sTI n éncMnYnkMupøictamTRmg;RtIekaNmaRt 12-TRmg;Giucs,:ÚNg;EsüléncMnYnkMupøic 13-RbmaNviFIcMnYnkMupøictamTRmg;Gics,:ÚNg;Esül 14-Gnuvtþn_cMnYnkMupøickñúgRtIekaNmaRt 15-Gnuvtþn_cMnYnkMupøickñúgsIVútcMnYnBit 16-Gnuvtþn_cMnYnkMupøickñúgFrNImaRt 17-bMElgcMnuckñúgbøg;kMupøic 18-)arIsg;énRbBn½æcMNuckñúgbøg;kMupøic - iv -

TMB&r 001 002 008 010 011 014 017 019 020 024 026 027 033 034 037 039 051 054

CMBUkTI2 kRmglMhat;eRCIserIs EpñkdMeNaHRsay

055 065

CMBUkTI3 lMhat;Gnuvtþn_

130

-v-

sikSaKNitviTüaedayxøÜnÉg

cMnYnkMupøic

CMBUkTI1

cMnYnkMupøic 1-niymn½y k> cMnYnnimμit plKuNéncMnYnBit c xusBIsUnünwg i ehAfacMnYnnimμit . i ehAfaÉktanimμitEdl i 2  1 b¤ i   1 . ]TahrN_ ³ 2i ,  5i , 23i , 3i , ... ehAfacMnYnnimμit . x> niymn½ycMnYnkMupøic cMnYnkMupiøcCacMnYnEdlmanrag z  a  i.b Edl a nig b CacMnYnBit . eKtagsMNMuéncMnYnkMupøiceday ¢ . a ehAfaEpñkBitén z  a  i .b EdlkMNt;tageday Re( z )  a . b ehAfaEpñknimμitén z  a  i .b EdlkMNt;tageday Im( z )  b . ]TahrN_1 ³ 1  2i ,  3  2i , 4  3i ,  1  4i , 5i ,  7i ehAfacMnYnkMupøic . ]TahrN_2³ rkEpñkBit nig Epñknimμitén z  3  2i ? EpñkBit nig Epñknimμitén z  3  2i KW Re(Z)  3 ; Im( z )  2 . eroberogeday lwm plÁún

- TMBr½1 -

sikSaKNitviTüaedayxøÜnÉg

cMnYnkMupøic

2-RbmaNviFIelIcMnYnkMupøic k> viFIbUkcMnYnkMupøic ]bmafaeKmanBIrcMnYnkMupøic z1  a  i.b nig z 2  c  i.d Edl a , b , c , d CacMnYnBit . eK)an z1  z 2  (a  i.b)  (c  id)  (a  c)  i(b  d) dUcenH z1  z 2  (a  c)  i.(b  d) . ]TahrN_ ³ eK[cMnYnkMupøic z1  3  2i nig z 2  7  5i . KNna z1  z 2 eK)an z1  z 2  ( 3  2i )  (7  5i )  (3  7)  (2i  5i ) dUcenH z1  z 2  4  3i . x> viFIdkcMnYnkMupøic ]bmafaeKmanBIrcMnYnkMupøic z1  a  i.b nig z 2  c  i.d Edl a , b , c , d CacMnYnBit . eK)an z1  z 2  (a  i.b)  (c  id)  (a  c)  i(b  d) dUcenH z1  z 2  (a  c)  i.(b  d) . ]TahrN_ ³ eK[cMnYnkMupøic z1  3  2i nig z 2  7  5i . KNna z1  z 2 eK)an z1  z 2  ( 3  2i )  (7  5i )  (3  7)  (2i  5i ) dUcenH z1  z 2  10  7i . eroberogeday lwm plÁún

- TMBr½2 -

sikSaKNitviTüaedayxøÜnÉg

cMnYnkMupøic

K> viFIKuNcMnYnkMupøic ]bmafaeKmanBIrcMnYnkMupøic z1  a  i.b nig z 2  c  i.d Edl a , b , c , d CacMnYnBit . eK)an z1  z 2  (a  i.b)(c  i.d)  ac  iad  ibc  i 2bd  ac  iad  ibc  bd  (ac  bd )  i(ad  bc )

dUcenH z1  z 2  (ac  bd)  i(ad  bc) . ]TahrN_ ³ eK[cMnYnkMupøic z1  2  i nig z 2  1  3i . KNna z1  z 2 eK)an z1 .z 2  (2  i )(1  3i )  2  6i  i  3i 2  5  5i dUcenH z1 .z 2  5  5i . X> viFIEckcMnYnkMupøic ]bmafaeKmanBIrcMnYnkMupøic z1  a  i.b nig z 2  c  i.d Edl a , b , c , d CacMnYnBit . z1 a  ib (a  ib )(c  id ) ac  iad  ibc  i 2bd eK)an z  c  id  (c  id)(c  id)  c2  i 2d 2 2 ac  iad  ibc  bd (ac  bd )  i(bc  ad )   2 2 c d c2  d 2 dUcenH zz1  ac2  bd2  i. bc2  ad2 . c d c d 2

eroberogeday lwm plÁún

- TMBr½3 -

sikSaKNitviTüaedayxøÜnÉg

cMnYnkMupøic

]TahrN_ ³ eK[cMnYnkMupøic z1  1  5i nig z 2  1  i . KNna zz1 eK)an zz1  115ii  ((115ii)()(11ii))  1  i151i  5  6 2 4i

2

2

dUcenH zz1  3  2i . 2

g> sV½yKuNén i eKmansV½yKuNén i dUcxageRkam ³ i1  i i 2  1 i 3  i 2 .i   i i 4  (i 2 )2  ( 1)2  1 i 5  i 4 .i  i i 6  i 5 .i  i .i  i 2  1 i 7  i 6 .i   i i 8  i 7 .i   i .i   i 2  1   CaTUeTA i4n  1 , i4n 1  i , i4n  2  1 , i4n  3  i

RKb; n  IN dUcenHcMnYn in esμInwgcMnYndEdl²KW i ,  1 ,  i nig 1 RKb; n  IN . eroberogeday lwm plÁún

- TMBr½4 -

sikSaKNitviTüaedayxøÜnÉg

cMnYnkMupøic

c> sV½yKuNéncMnYnkMupøic eKmansV½yKuNéncMnYnkMupøicdUcxageRkam ³ (a  ib )2  a 2  b 2  i .2ab (a  ib )3  (a 3  3ab 2 )  i( 3a 2b  b 3 ) (a  ib )4  (a4  6a 2b 2  b4 )  i(4a 3b  4ab 3 )   n

(a  ib )   C(n, k ) an  k bk .i k n

k 0

Edl c(n, k )  k! (nn! k )! . ]TahrN_ ³ KNna (1  3i )2 , (2  i )3 nig (1  2i )4 . eK)an ³ (1  3i )2  1  6i  9  8  6i ( 2  i )3  8  12i  6  i  2  11i (1  2i )4  1  8i  24  32i  16  7  24i

q> kMupøicesμIKña ]bmafa z1  a  ib nig z 2  c  i.d Edl a, b, c, d CacMnYnBit .  ac z1  z 2    bd

eroberogeday lwm plÁún

- TMBr½5 -

sikSaKNitviTüaedayxøÜnÉg

cMnYnkMupøic

dUcenHcMnYnkMupøicBIresμIKñaluHRtaEtEpñkBitesμIKña nig EpñknimμitesμIKña . ]TahrN_ ³ eK[ z1  2  3  4i nig z 2    9  8i cUrkMNt;BIrcMnYnBit  nig  edIm,I[ z1  z 2 ? kMNt;  nig  ³

 2  3    9 2  3  4i    9  8i   4  8 eKTaj)an   2 ,   3 .

C> KNnab¤skaeréncMnYnkMupøic ]bmafaeKmancMnYnkMupøic z  a  i.b Edl a , b CacMnYnBit edIm,IKNnab¤skaerén z eKRtUvGnuvtþn_dUcxageRkam ³ tag w  x  i.y Cab¤skaerén z  a  i.b ¬ x , y CacMnYnBit¦ eK)an w 2  z ( x  i .y )2  a  i .b ( x 2  y 2 )  i( 2xy )  a  ib x2  y 2  a eKTaj)an  2xy  b 

edaHRsayRbBn½æsmIkarenHeK)anKUcemøIy (x, y )  {(1 , 1 ); (2 ,  2 )} dUcenH w1  1  i1 ; w 2  2  i 2 .

eroberogeday lwm plÁún

- TMBr½6 -

sikSaKNitviTüaedayxøÜnÉg

cMnYnkMupøic

]TahrN_1 ³ KNnab¤skaeréncMnYnkMupøic z  21  20i tag w  x  i.y Cab¤skaerén z  21  20i ¬ x , y CacMnYnBit¦ eK)an w 2  z ( x  i .y )2  21  20i ( x 2  y 2 )  i( 2xy )  21  20i  x 2  y 2  21 eKTaj)an  2xy  20 

bnÞab;BIedaHRsayRbBn½æsmIkarenHeK)anKUcemøIy ³ x  5 , y  2 b¤ x  5 , y  2 dUcenH w1  5  2i ; w 2  5  2i . ]TahrN_2 ³ KNnab¤skaeréncMnYnkMupøic z  21  20i tag w  x  i.y Cab¤skaerén z  8  6i ¬ x , y CacMnYnBit¦ eK)an w 2  z ( x  i .y )2  8  6i ( x 2  y 2 )  i( 2xy )  8  6i  x 2  y 2  8 eKTaj)an  2xy  6  bnÞab;BIedaHRsayeK)anKUcemøIy ³ x  1 , y  3

dUcenH w1  1  3i

; w 2   1  3i

eroberogeday lwm plÁún

b¤ x  1 , y  3

. - TMBr½7 -

sikSaKNitviTüaedayxøÜnÉg

cMnYnkMupøic

3-cMnYnkMupøicqøas; k> niymn½y cMnYnkMupøicqøas;éncMnYnkMupøic z  a  i.b , a; b  IR KWCacMnYnkuMpøicEdl kMNt;tageday z  a  i.b . ]TahrN_ ³ cMnYnkMupøicqøas;én z  4  3i KW z  4  3i . x> lkçN³ !> (z1  z 2 )  z1  z 2 @> (z1 .z 2 )  z1 .z 2  z1  z1 #>  z   z  2 2

sRmaybBa¢ak; tag z1  a  ib nig z 2  c  id Edl a , b , c , d CacMnynBit . eK)an z1  a  i.b nig z 2  c  i.d man z1  z 2  (a  c)  i(b  d) enaH z1  z 2  (a  c)  i(b  d) ehIy z1  z 2  a  ib  c  id  (a  c)  i(b  d) dUcenH (z1  z 2 )  z1  z 2 . eKman z1z 2  (a  ib)(c  id)  (ac  bd)  i(ad  bc) eK)an z1 .z 2  (ac  bd )  i(ad  bc) ehIy z1 .z 2  (a  ib)(c  id)  (ac  id)  i(ad  bc) eroberogeday lwm plÁún

- TMBr½8 -

sikSaKNitviTüaedayxøÜnÉg

cMnYnkMupøic

dUcenH (z1 .z 2 )  z1 .z 2 . ib (a  ib )(c  id ) ac  bd bc  ad  i .   2 eKman zz1  ac  id 2 2 2 (c  id )(c  id ) c d

2

c d

eK)an  zz1   ac2  bd2  i. bc2  ad2 ehIy

c d  2 c d z1 a  ib (a  ib )(c  id ) ac  bd bc  ad    2  i . z 2 c  id (c  id )(c  id ) c  d 2 c2  d 2

 z1  z 1 dUcenH  z   z  2 2

.

K> kenSamEpñkBit nig EpñknimμitCaGnuKmn_én z nig z ]bmafaeKman z  a  ib enaH z  a  ib Edl a; b CacMnYnBit . eK)an z  z  a  ib  a  ib  2a enaH a  z 2 z ehIy z  z  a  ib  a  ib  2ib enaH b  z 2i z

dUcenH Re(z )  z 2 z nig Im( z )  z 2i z . -ebI Re(z )  0 enaH z   z naM[ z CacMnYnnimμit . -ebI Im( z )  0 enaH z  z naM[ z CacMnYnBit .

eroberogeday lwm plÁún

- TMBr½9 -

sikSaKNitviTüaedayxøÜnÉg

cMnYnkMupøic

4-Gnuvtþn_cMnYnkMupøickñúgdMeNaHRsaysmIkardWeRkTIBIr ]bmafaeKmansmIkardWeRkTIBIr az 2  bz  c  0 Edl a  0 , a, b , c CacMnYnBit . DIsRKImINg;énsmIkarKW   b2  4ac -ebI   0 smIkarmanb¤sBIepSgKñaCacMnYnBitKW ³ z1 

b  b  ; z2  2a 2a

-ebI   0 smIkarmanb¤sDúbCacMnYnBitKW z1  z 2   2ba -ebI   0 smIkarmanb¤sBIepSgKñaCacMnYnkMupøicqøas;KñaKW ³ z1 

bi || bi || ; z2  2a 2a

]TahrN_ ³ edaHRsaysmIkar 2z 2  6z  5  0 DIsRKImINg;énsmIkar   36  40  4  4i 2 eKTajb¤s z1  6 4 2i  32  i. 12 ; z 2  6 4 2i  32  i. 12 dUcenHsmIkarmanb¤dBIrepSgKñaCacMnYnkMupøicqøas;KñaKW ³ 1 3 1 3 z1   i . ; z 2   i . . 2 2 2 2

eroberogeday lwm plÁún

- TMBr½10 -

sikSaKNitviTüaedayxøÜnÉg

cMnYnkMupøic

5-cMnYnkMupøickñúgbøg; k> kartagcMnYnkMupøickñúgbøg; kñúgtRmuyGrtUNrem (xoy) eKGactagcMnYnkMupøic z  a  i.b ; a, b  IR edaycMNuc M mYymankUGredaen (a, b) . eKfa M CacMnucrUbPaBéncMnYnkMupøic z  a  ib ehIy z ehAfaGaPicén cMnuc M(a, b) EdleKkMNt;sresr M(z ) . dUcKñaEdr eKk¾GactagcMnYnkMupøic z  a  i.b ; a, b  IR edayviucTr½  

 

u  OM  (a, b )

eKfa viucTr½

 

u  

u

.

CavIucTr½rUbPaBéncMnYnkMupøic z  a  ib ehIy z ehAfaGaPicén 

EdleKkMNt;sresr u (z ) . y b

M

o

a

eroberogeday lwm plÁún

x

- TMBr½11 -

sikSaKNitviTüaedayxøÜnÉg

cMnYnkMupøic

x> viucTr½rUbPaBénplbUkcMnYnkMupøickñúgbøg;kMupøic kñúgtRmuyGrtUNrem (xoy) ]bmafaeKmancMnYnkMupøicBIr z1 nig z 2 ehIytag M1 nig M 2 CarUbPaBén z1 nig z 2 .  

eK)an OM1 nig

 

OM 2

CaviucTr½rUbPaBén z1 nig z 2 .

y

M M1

M2

O

x  

 

 

eKman z1  z 2  OM1  OM 2  OM dUcenHrUbPaBén z1  z 2 KWCaviucTr½Ggát;RTUgénRbelLÚRkam OM1MM 2 .

eroberogeday lwm plÁún

- TMBr½12 -

sikSaKNitviTüaedayxøÜnÉg

cMnYnkMupøic

K> viucTr½rUbPaBénpldkcMnYnkMupøickñúgbøg;kMupøic kñúgtRmuyGrtUNrem (xoy) ]bmafaeKmancMnYnkMupøicBIr z1 nig z 2 ehIytag M1 nig M 2 CarUbPaBén z1 nig z 2 .  

eK)an OM1 nig

 

OM 2

CaviucTr½rUbPaBén z1 nig z 2 . y

M2 M1

x

O

x'

M

M2'

y'  

 

 

eK)an z1  z 2  z1  ( z 2 )  OM1  OM'2  OM dUcen rUbPaBén z1  z 2 KWCaviucTr½Ggát;RTUgénRbelLÚRkam OM1MM'2 . eroberogeday lwm plÁún

- TMBr½13 -

sikSaKNitviTüaedayxøÜnÉg

cMnYnkMupøic

X> viucTr½rUbPaBénplKuNcMnYnBit nwg cMnYnkMupøickñúgbøg;kMupøic ³ y M'

M

x

O

]bmafa M nig M' CacMnucrUbPaBén z nig z , (   0)  

 

 

rUbPaBén  .z KW OM' Edl OM'   OM . 6-m:UDúléncMnYnkMupøic k> niymn½y kñúgtRmuyGrtUNrem (xoy) eKyk M(a, b) CarUbPaBén z  a  ib . rgVas; OM ehAfam:UDúlén z  a  ib . eKkMNt;tagm:UDúlén z  a  ib eday | z | b¤ r EdlGacKNna)antam rUbmnþ | z | r  OM  a2  b2 . eroberogeday lwm plÁún

- TMBr½14 -

sikSaKNitviTüaedayxøÜnÉg

cMnYnkMupøic

y M

b

|z| P o

a

x

kñúgRtIekaNEkg OMP eKman OM 2  MP 2  OP 2 eday OP  a , MP  b eK)an OM 2  a2  b2 b¤ OM  a2  b2 . dUcenH | z | OM  a2  b2 . x> lkçN³ !> | z || z | @> z.z | z |2 #> | z1 .z 2 || z1 | . | z 2 | $> zz1  || zz1 || 2 n

%> | z

2 n

|| z |

eroberogeday lwm plÁún

- TMBr½15 -

sikSaKNitviTüaedayxøÜnÉg

cMnYnkMupøic

K> vismPaBRtIekaN RKb;cMnYnkMupøic z1 nig z 2 eKman | z1  z 2 | | z1 |  | z 2 | sRmay ³ tag z1  x  iy nig z 2  u  iv eKman z1  z 2  (x  u)  i(y  v) eK)an | z1  z 2 | (x  u)2  (y  v)2 nig | z1 |  | z 2 | x2  y 2  u2  v 2 eday | z1  z 2 |2  x2  y 2  u2  v 2  2(xu  yv ) nig | z1 |  | z 2 |2  x2  y 2  u2  v 2  2 (x2  y 2 )(u2  v 2 ) eK)an ³



| z1 |  | z 2 |2  | z1  z 2 |2  2 (x2  y 2 )(u2  v 2 )  (xu  yv) kenSam | z1 |  | z 2 |2  | z1  z 2 |2  0 luHRtaEt



( x 2  y 2 )(u 2  v 2 )  ( xu  yv )2  0 x 2u 2  x 2 v 2  u 2y 2  v 2 y 2  x 2u 2  2xyuv  y 2 v 2  0 x 2 v 2  2xyuv  u 2y 2  0

Bit dUcenH | z1  z 2 | | z1 |  | z 2 | . ( xv  uy )2  0

eroberogeday lwm plÁún

- TMBr½16 -

sikSaKNitviTüaedayxøÜnÉg

cMnYnkMupøic

7-GaKuym:g;éncMnYnkMupøic kñúgtRmuyGrtUNrem (xoy) eKyk M(a, b) CarUbPaBén z  a  ib . y M

b

 o

P a

x

   

mMuEdlpÁúMeday ( Ox , OM ) ehAfaGaKuym:g;én z  a  i.b . eKtag  b¤ Arg( z ) CaGaKuym:g;én z  a  i.b . kñúgRtIekaNEkg OMP eKman ³ r 2  OM 2  a 2  b 2 b¤ r  a 2  b 2 ¬RTwsþIbTBItaKr½¦ OP a MP b cos    nig sin    . OM r OM r edIm,IrkGaKuym:g;én z  a  i.b eKedaHRsaysmIkar ³ a b cos   nig sin   eK)an Arg( z )    2k , k  Z . r r eroberogeday lwm plÁún

- TMBr½17 -

sikSaKNitviTüaedayxøÜnÉg

cMnYnkMupøic

]TahrN_ 1 ³ rkGaKuym:g;éncMnYnkMupøic z  2 3  2i tamrUbmnþ r  | z |  a2  b2  (2 3 )2  22  4 a 2 3 3 b 2 1 ni g cos     sin     r 4 2 r 4 2

dUcenHGaKuym:g;én z KW Arg(z )  6  2k ; k  Z ]TahrN_ 2 ³ rkGaKuym:g;éncMnYnkMupøic z  1  i 3 tamrUbmnþ r  | z |  a2  b2  a 1 b 3 cos     nig sin    r 2 r 2

( 1)2  ( 3)2  2

dUcenHGaKuym:g;én z KW Arg(z )  23  2k ; k  Z TahrN_ 3 ³ rkGaKuym:g;éncMnYnkMupøic z  2  i 2 tamrUbmnþ r  | z |  a2  b2  2  2  2 a 2 b 2 ni g cos    sin     r 2 r 2

dUcenHGaKuym:g;én z KW Arg(z )   4  2k ; k  Z

eroberogeday lwm plÁún

- TMBr½18 -

sikSaKNitviTüaedayxøÜnÉg

cMnYnkMupøic

8-TRmg;RtIekaNmaRténcMnYnkMupøic cMnYnkMupøic z  a  i.b ehAfaTRmg;BICKNit . eKGacsresrTRmg;fμImYy eTotdUcxageRkam ³ eKman r  a2  b2 ehAfam:Dúlén z  a  i.b a b cos   nig sin   Edl  ehAfaGaKuym:g;én z . r r eK)an z  a  i.b  r( ar  i. br )  r(cos   i. sin ) dUcenH z  r(cos   i. sin ) ehAfaTRmg;RtIekaNmaRtén z . ]TahrN_ 1 ³ cUrsresr z  1  i 3 CaragRtIekaNmaRt . eKman r  12  ( 3 )2  2 eK)an z  2( 12  i. 23 )  2(cos 3  i. sin 3 ) . ]TahrN_ 2 ³ cUrsresr z  2 3  2i CaragRtIekaNmaRt . eKman r  (2 3 )2  (2)2  4 eK)an z  4( 23  i. 12 )  4( cos 6  i. sin 6 ) .   z  4[cos(   )  i . sin(   )] 6 6 5 5 z  4(cos  i sin ) 6 6

eroberogeday lwm plÁún

- TMBr½19 -

sikSaKNitviTüaedayxøÜnÉg

cMnYnkMupøic

9-RbmaNviFIelIcMnYnkMupøictamTRmg;RtIekaNmaRt k> plKuNcMnYnkMupøictamTRmg;RtIekaNmaRt RTwsþIbT ³ ]bmafaeKmancMnYnkMupøic z1  r1 (cos   i. sin ) nig z 2  r2 (cos   i sin  ) Edl r1  0 , r2  0 eK)an z1 .z 2  r1r2[cos(   )  i sin(   )] . sRmaybBa¢ak; eK)an z1z 2  r1(cos   i. sin ).r2 (cos   i sin ) z1z2  r1r2[(cos cos  sin sin)  i(sin cos  sin cos)

dUcenH z1 .z 2  r1r2[cos(   )  i sin(   )] . ]TahrN_ eK[cMnYnkMu[pøic 2 2   z1  2(cos  i sin ) nig z 2  3(cos  i sin ) 15 5 5 15 KNna z1 .z 2 eK)an z1z 2  6[cos( 5  215 )  i sin( 5  215 )]

5 5   i sin  )  6(cos  i . sin ) 15 15 3 3   z1z 2  6(cos  i . sin ) . 3 3  6(cos

dUcenH

eroberogeday lwm plÁún

- TMBr½20 -

sikSaKNitviTüaedayxøÜnÉg

cMnYnkMupøic

x> plKuNEckcMnYnkMupøictamTRmg;RtIekaNmaRt RTwsþIbT ³ ]bmafaeKmancMnYnkMupøic z1  r1 (cos   i. sin ) nig z 2  r2 (cos   i sin  ) Edl r1  0 , r2  0 eK)an zz1  rr1 [cos(  )  i sin(  )] . 2

2

sRmaybBa¢ak;   i sin  eK)an zz1  rr1 . cos cos   i sin  2

2



r1 (cos   i. sin )(cos   i sin ) . r2 (cos   i sin )(cos   i sin )



r1 (cos  cos   sin  sin )  i(sin  cos   sin  cos ) . r2 cos 2   sin 2 



r1 cos(  )  i sin(  ) r2

dUcenH zz1  rr1 [cos(  )  i sin(  )] . 2

2

K> sV½yKuNTI n éncMnYnkMupøictamTRmg;RtIekaNmaRt ³ RTwsþIbT ³ RKb;cMnYnBit  nig r  0 eK)an ³ z n  [r(cos   i . sin  )]n  r n (cos n  i . sin n )

Edl n CacMnYnKt;rWLaTIhV . eroberogeday lwm plÁún

- TMBr½21 -

sikSaKNitviTüaedayxøÜnÉg

cMnYnkMupøic

sRmaybBa¢ak; ³ ramrUbmnþ z1 .z 2  r1r2[cos(  )  i sin(   )] eK)anCabnþbnÞab;dUcxageRkam ³ z 2  z .z  r .r[cos(   )  i sin(    )]  r 2 (cos 2  i sin 2 ) z 3  z 2 .z  r 2 .r [cos( 2   )  i sin( 2   )]  r 3 (cos 3  i . sin 3 )   ]bmafa zn  rn (cos n  i. sin n) Bit

eK)an zn1  zn .z  rn .r[cos(n  )  i sin(n  )]  r n 1[cos(n  1)  i sin(n  1)] Bit dUcenH zn  rn (cos n  i. sin n) . X> rUbmnþdWmr½ eKman zn  [r(cos   i. sin )]n  rn (cos n  i. sin n) eK)an rn (cos   i. sin )n  rn (cos n  i. sin n) dUcenH (cos   i. sin )n  cos n  i. sin n ¬ehAfarUbmnþdWmr½ ¦ .

eroberogeday lwm plÁún

- TMBr½22 -

sikSaKNitviTüaedayxøÜnÉg

cMnYnkMupøic

]TahrN_ ³ eK[cMnYnkMupøic z  cos 49  i. sin 49 cUrsresr w 

z2 1 z

3

CaragRtIekaNmaRt .

tamrUbmnþdWmr½eKman z 2  (cos 49  i sin 49 )2  cos 89  i sin 89 4 4 3 4 4 nig z  (cos 9  i sin 9 )  cos 3  i sin 3 8 8 cos  i . sin 9 9 eK)an w  4 4 1  cos  i . sin 3 3 8 8  i . sin cos 9 9  2 2 2  2i sin cos 2 cos 2 3 3 3 8 8  i . sin cos 1 9 9  . 2 2 2  i sin cos 2 cos 3 3 3 8 2 8 2  [cos(  )  i sin(  )] 9 3 9 3 11 11 2 2   (cos  i sin )  cos  i sin 9 9 9 9 11 11 dUcenH w  cos 9  i. sin 9 . 3

eroberogeday lwm plÁún

- TMBr½23 -

sikSaKNitviTüaedayxøÜnÉg

cMnYnkMupøic

10-bMElgvilCMuvijKl;tRmuyénbøg;kMupøic eKmancMnYnkMupøic w  cos   i. sin  . ebI M' (z' ) CarUbPaBén M(z ) tambEmøgvilp©it O nig mMu  enaH eK)an z'  w. z  (cos   i sin  ) .z . y

M'

 O

M x

]TahrN_ 1 ³ enAkñúgbøg;kMupøic (xoy) eK[ M CacMnucrUbPaBén z  3  i . cUrkMNt; z' edaydwgfa M' (z' ) CarUbPaBén M tambEmøgvilp©it O  nigmMu   12 . eroberogeday lwm plÁún

- TMBr½24 -

sikSaKNitviTüaedayxøÜnÉg

cMnYnkMupøic

ebI M' (z' ) CarUbPaBén M(z ) tambEmøgvilp©it O nigmMu   12 enaHeK)an

   i sin ) z 12 12 3 1   eday z  3  i  2( 2  i. 2 )  2(cos 6  i. sin 6 ) eK)an z'  2[cos(12  6 )  i sin(12  6 )]   dUcenH z'  2(cos 4  i sin 4 ) . ]TahrN_ 2 ³ enAkñúgbøg;kMupøic (xoy) eK[ M CacMnucrUbPaBén z  1  i 3 z'  (cos

.

cUrkMNt; z' edaydwgfa M' (z' ) CarUbPaBén M tambEmøgvilp©it O nigmMu   23 . ebI M' (z' ) CarUbPaBén M(z ) tambEmøgvilp©it O nigmMu

2  3

enaHeK)an

2 2  i sin ) z 3 3 eday z  1  i 3  2( 12  i. 23 )  2[cos( 3 )  i. sin(  3 )] eK)an z'  2[cos( 23  3 )  i sin( 23  3 )] dUcenH z'  2(cos 3  i sin 3 ) . z'  (cos

eroberogeday lwm plÁún

- TMBr½25 -

sikSaKNitviTüaedayxøÜnÉg

cMnYnkMupøic

11-b¤sTI n éncMnYnkMupøictamTRmg;RtIekaNmaRt ]bmafaeKmancMnYnkMupøic z  r(cos   i sin  ) tag w  (cos   i sin ) Cab¤sTI n én z enaH wn  z eK)an n (cos n  i sin n)  r(cos   i sin  ) n  r  eKTaj cos(n)  cos  naM[ sin(n )  sin   dUcenHb¤sTI n én z kMNt;eday ³

  n r     2k     n

    2k     2k   w k  n r cos   i . sin  n n      Edl k  0 , 1 , 2 , ...., n  1 .

]TahrN_ ³ KNnab¤sTIbIén z  4 2  i.4 2 eKman z  4 2  i.4 2  8( 22  i 22 )  8(cos 4  i sin 4 )

eKTaj r  8 ,   4 . tamrUbmnþb¤sTI # én z kMNt;eday ³

    8k     8k   w k  2cos  i sin    , k  0 , 1 , 2 .  12     12  dUcenH w 0  2(cos 12  i sin 12 ) ; w1  2(cos 34  i sin 34 ) nig w 2  2(cos 1712  i sin 1712 ) .

eroberogeday lwm plÁún

- TMBr½26 -

sikSaKNitviTüaedayxøÜnÉg

cMnYnkMupøic

12-TRmg;Giucs,:ÚNg;EsüléncMnYnkMupøic k> rUbmnþGWEl (Euler's formula) cMeBaHRKb;cMnYnBit x eK)an eix  cos x  i sin x Edl e  2.71828... CaeKalelakarIteEB . rUbmnþGWElenHenAEtBitcMeBaH x CacMnYnkMupøick¾eday . sRmaybBa¢ak; -RsaybBa¢ak;edayeRbIedrIev ³ tagGnuKmn_ f ¬GacCaGnuKmn_kMupøic¦énGefr x kMNt;eday f (x) 

cos x  i sin x

eix (  sin x  i cos x )eix  ieix (cos x  i sin x ) eK)an f ' (x)  e2ix eix (  sin x  i cos x  i cos x  i 2 sin x )  e 2ix  sin x  i cos x  i cos x  sin x 0   ix  0 ix e e naM[ f (x) CaGnuKmn_efrRKb; x . eK)an f (x)  f (0)  cos 0 0i sin 0  1 b¤ f (x)  cos x ixi sin x  1 e e dUcenH eix  cos x  i sin x .

eroberogeday lwm plÁún

- TMBr½27 -

sikSaKNitviTüaedayxøÜnÉg

cMnYnkMupøic

-RsaybBa¢ak;edayeRbIsmIkarDIepr:g;EsüllMdab;TImYy ³ eKtagGnuKmn_ g(x)  cos x  i. sin x eKman g' (x)   sin x  i cos x KuNGgÁTaMgBIrnwg i eK)an i.g' (x)  i sin x  csox eK)an g(x)  ig' (x)  0 b¤ g' (x)  i . g(x)  0 CasmIkarDIepr:g;EsüllMdab; I . eK)ancemøIyTUeTAénsmIkarenHKW g(x)  k eix ebI x  0 enaH g(0)  k Et g(0)  cos 0  i. sin 0  1 enaH k  1 ehIyeK)an g(x)  eix . dUcenH eix  cos x  i sin x . -RsaybBa¢ak;edayeRbIsmIkarDIepr:g;EsüllMdab;TIBIr ³ eRCIserIsGnuKmn_ h(x)  eix eKman h' (x)  i.eix nig h' ' (x)  i 2eix  eix eK)an h' ' (x)  h(x)  eix  eix  0 CasmIkarDIepr:g;EsüllIenEG‘Gum:UEsn lMdab;TIBIr . smIkarDIepr:g;EsüllIenEG‘lMdab;BIrenHmancemøIylIenEG‘ÉkraCülIenEG‘cMnYnBIr EdlepÞógpÞat;vaKW h1(x)  cos x nig h2 (x)  sin x . bnSMlIenEG‘éncemøIy cMeBaHsmIkarDIepr:g;EsülGUm:UEsn k¾CacemøIymYypgEdr . eroberogeday lwm plÁún

- TMBr½28 -

sikSaKNitviTüaedayxøÜnÉg

cMnYnkMupøic

dUcenHcemøIyTUeTAénsmIkarKW h(x)  A cos x  B sin x Edl A nig B CaBIrcMnYnefrEdlGacrk)antam h(0)  A  ei0  1 nig h' (0)  B  iei0  i eRBaH h' (x)   A sin x  B cos x . ehtuenHeK)an h(x)  cos x  i sin x . dUcenH eix  cos x  i sin x . -RsaybBa¢ak;edayeRbIesrIétlr½³ rUbmnþes‘rIétlr½cMeBaHGnuKmn_bI ex , cos x nig sin x KW ³ xn x x2 x3   .....   ..... e  1  n! 1! 2! 3! x

2n x 2 x4 x6 n x    .....  ( 1)  ...... cos x  1  ( 2n )! 2! 4! 6! 2n  1 x 3 x5 x7 n x sin x  x     ....  ( 1)  .... 3! 5! 7! ( 2n  1)!

edayCMnYs x eday ix kñúges‘TaMgbIenHeKTaj)an eix  cos x  i sin x . x> TRmg;Giucs,:ÚNg;Esül RKb;cMnYnkMupøic z  a  i.b Edl a , b CacMnYnBitGacsresrCaTRmg;mYyfμIeTot KW z  r ei Edl r  a2  b2 , cos   ar , sin   br . TRmg; z  r ei ehAfaTRmg;Gics,:ÚNg;Esülén z  a  i.b . eroberogeday lwm plÁún

- TMBr½29 -

sikSaKNitviTüaedayxøÜnÉg

cMnYnkMupøic

K> TMnak;TMngCamYyGnKmn_ u RtIekaNmaRt ³ cMeBaHRKb;cMnYnBit x eKman eix  cos x  i sin x (1) CMnYs x eday  x eK)an e ix  cos x  i sin x (2) bUksmIkar (1) nig (2) eKTaj)an eix  e ix  2 cos x

eix  e ix eKTaj cos x  2 . dksmIkar (1) nig (2) eKTaj)an eix  e ix  2i sin x

eix  e ix eKTaj sin x  2i . eix  e ix eix  e ix dUcenH cos x  2 ; sin x  2i ¬ rUbmnþenHBitpgEdrcMeBaH x CacMnYnkMupøic ¦

.

X> TMnak;TMngCamYyGnuKmn_GIEBbUlik³

eix  e ix eix  e ix eKman cos x  2 ; sin x  2i CMnYs x eday i.x eK)an ³

ex  e x ex  e x cos(ix )   cosh x nig sin( ix )  i  i sinh x 2 2 ex  e x ex  e x mü:ageToteKman cosh x  2 nig sinh x  2 eK)an ³ eix  e ix eix  e ix cosh(ix )   cos x , sinh( ix )   i sin x . 2 2

eroberogeday lwm plÁún

- TMBr½30 -

sikSaKNitviTüaedayxøÜnÉg

cMnYnkMupøic

]TahrN_1 ³ sresr z  2  2i 3 CaTRmg;Giucs,:ÚNg;Esül ? eKman r  22  (2 3 )2  4 eK)an z  4( 12  i

3   )  4(cos  i sin )  4 e 2 3 3 i

i

 3

.

 4

]TahrN_2³ sresr z  1  e CaragGics,:ÚNg;Esül ? eKman

z

 i  1 e 4

  i i  e 8 (e 8 

 i8 dUcenH z  2 cos 8 .e ]TahrN_3³ KNna ii ?

 i e 8)

eday cos 8  e

 8

e 2

i

 8

. 

i   eKman i  cos 2  i. sin 2  e 2 i   i   eK)an ii   e 2   e 2 .   ]TahrN_4³ edaHRsaysmIkar cos x  2

kñúgsMNMukMupøic ?

eix  e ix tamrUbmnþGWEleKman cos x  2 eix  e ix eK)an 2  2 b¤ e2ix  4eix  1  0

eroberogeday lwm plÁún

i

tag t  eix - TMBr½31 -

sikSaKNitviTüaedayxøÜnÉg

cMnYnkMupøic

eK)ansmIkar t 2  4t  1  0 '  4  1  3 eKTajb¤s t1  2  3 ; t 2  2  3 cMeBaH t  2  3 eK)an eix  2  3 enaH ix  ln( 2  b¤ x  i ln( 2  3 ) . cMeBaH t  2  3 eK)an eix  2  3 enaH ix  ln( 2  b¤ x  i ln( 2  3 ) . ]TahrN_5³ edaHRsaysmIkar sin x  3 kñúgsMNMukMupøic ? eix  e ix tamrUbmnþGWEleKman sin x  2i eix  e ix eK)an 2i  3 b¤ e2ix  6eix  1  0 eK)ansmIkar t 2  6t  1  0 , '  9  1  10

3)

3)

tag t  eix

manb¤s t1  3  10 , t 2  3  10 . -cMeBaH t  3  10 eK)an eix  3  10 enaH x  i ln( 3  -cMeBaH t  3  10  (3  10 ) eK)an eix  (3  10 )  (3  10 )ei  ei  ln( 3 10 ) eKTaj ix  i  ln( 3  10 ) b¤ x    i ln( 3  10 ) . dUcenH x  i ln( 3  10 ) , x    i ln( 3  10 ) .

eroberogeday lwm plÁún

10 )

- TMBr½32 -

sikSaKNitviTüaedayxøÜnÉg

cMnYnkMupøic

13-RbmaNviFIcMnYnkMupøictamTRmg;Gics,:ÚNg;Esül k> RbmaNviFIKuNcMnYnkMupøictamTRmg;Gics,:ÚNg;Esül eKmancMnYnkMupøic z  r ei nig w   ei Edl r  0 ;   0 ehIy  ,  CacMnYnBit . eK)an z.w  r. ei(   ) . x> RbmaNviFIEckcMnYnkMupøictamTRmg;Gics,:ÚNg;Esül eKmancMnYnkMupøic z  r ei nig w   ei Edl r  0 ;   0 ehIy  ,  CacMnYnBit . eK)an wz  r ei(   ) . K> sV½yKuNcMnYnkMupøictamTRmg;Gics,:ÚNg;Esül eKmancMnYnkMupøic z  r ei cMeBaHRKb;cMnYnKt;rWLaTIhV n eK)an ³ n z n  r ei   r n ein . ]TahrN_ ³ eK[cMnYnkMupøic z  2e KNna z.w nig wz eK)an z.w 

  i(  ) 6 e 6 12

eroberogeday lwm plÁún

 6e

i

i

 4

 6

nig nig

w  3e

i

 12

.

 



z 2 i ( 6  12 ) 2 i 12  e  e w 3 3

- TMBr½33 -

sikSaKNitviTüaedayxøÜnÉg

cMnYnkMupøic

14-Gnuvtþn_cMnYnkMupøickñúgRtIekaNmaRt k> rUbmnþmMuDúb cMeBaHRKb;cMnYnBit x eKman

eix  e ix eix  e ix cos x  ; sin x  2 2i 2

2ix  2ix  eix  e ix  e e  2 2 eK)an cos x   2   4   1 1 e2ix  e i 2 x 1 1 2 cos x  .   cos 2x  2 2 2 2 2 eKTaj cos 2x  2 cos2 x  1 .

e2ix  e 2ix eix  e ix eix  e ix ehIy sin 2x  2i  2. 2i . 2 dUcenH sin 2x  2 sin x cos x .

x> rUbmnþmMuRTIb 3

 eix  e ix  3 eKman cos x   2    e3ix  e 3ix  3(eix  e ix )  8 2 cos 3x  6 cos x cos 3x  3 cos x   8 4 eKTaj)an cos 3x  4 cos3 x  3 cos x .

eroberogeday lwm plÁún

- TMBr½34 -

sikSaKNitviTüaedayxøÜnÉg

cMnYnkMupøic 3

 eix  e ix  3 ehIy sin x   2i    (e 3ix  e  3ix )  3(eix  e  ix )   8i 2i sin 3x  6i sin x   8i sin 3x  3 sin x  4 eKTaj)an sin 3x  3 sin x  4 sin3 x .

K> rUbmnþplbUk nig pldkénmMuBIr eKman ei(a b )  eia .eib RKb;cMnYnBit a nig b eday eia .eib  (cos a  i sin a)(cos b  i sin b) eiaeib  (cosa cosb  sina sinb)  i(sina cosb  sinb cosa)

ehIy ei(a b )  cos(a  b)  i sin(a  b) dUcenH cos(a  b)  cos a cos b  sin a sin b nig sin(a  b)  sin a cos b  sin b cos a . mü:ageTot ei(ab)  eia .e ib RKb;cMnYnBit a nig b eday eia .e ib  (cos a  i sin a)(cos b  i sin b)

eiaeib  (cosa cosb  sina sinb)  i(sina cosb  sinb cosa)

ehIy ei(ab )  cos(a  b)  i sin(a  b) eroberogeday lwm plÁún

- TMBr½35 -

sikSaKNitviTüaedayxøÜnÉg

cMnYnkMupøic

dUcenH cos(a  b)  cos a cos b  sin a sin b nig sin(a  b)  sin a cos b  sin b cos a . X> rUbmnþbMElgBIplKuNeTAplbUk

eia  e  ia eib  e ib cos a cos b  . 2 2 ei ( a  b )  ei ( a  b )  e  i ( a  b )  e  i ( a  b )  4 1  ei ( a  b )  e  i ( a  b ) ei ( a  b )  e  i ( a  b )      2  2 2 

1 cos(a  b)  cos(a  b ) 2 dUcenH cos a cos b  12 cos(a  b)  cos(a  b) eia  e ia eib  e ib sin a sin b  . 2i 2i ei ( a  b )  ei ( a  b )  e  i ( a  b )  e  i ( a  b )  4i 2 1  ei ( a  b )  e  i ( a  b ) ei ( a  b )  e  i ( a  b )      2  2 2  1   cos(a  b )  cos(a  b ) 2 dUcenH sin a sin b  12 cos(a  b)  cos(a  b) 

eroberogeday lwm plÁún

- TMBr½36 -

sikSaKNitviTüaedayxøÜnÉg

cMnYnkMupøic

15-Gnuvtþn_cMnYnkMupøickñúgsIVútcMnYnBit cMeBaHsIVúténcMnYnBit (an ) EdlepÞógpÞat;TMnak;TMngkMeNIn ³ an  2  pan  1  qan  0 Edl p , q CacMnYnBit smIkarsmÁal;rbs;sIVútenHKW z 2  pz  q  0 kñúgkrNI   p2  4q  0 smIkarmanb¤sBIrCacMnYnkMupøicqøas;Kña KW z1 nig z 2 . kñúgkrNIenHedIm,IKNna an eKRtUvGnuvtþn_dUcteTA ³ tagsIVútCMnYy zn  an 1  z1an rYcRsayfa (zn ) CasIVútFrNImaRt éncMnYnkMupøicmYy. KNna zn rYcTajrk an . ]TahrN_ eKmansIVúténcMnYnBit (an ) kMNt;eday ³ a0  0 , a1  1 nig an  2  an 1  an Edl n  0 , 1 , 2 ,... cUrKNna an CaGnuKmn_én n smIkarsmÁal;énsIVút an 2  an1  an KW ³ r 2  r  1 b¤ r 2  r  1  0 ;   1  4  3  3i 2 manb¤s r1  1  2i 3 ; r2  1  2i 3 tagsIVútCMnYy zn  an1  1  2i eK)an zn1  an 2  1  2i eroberogeday lwm plÁún

3

3

an

an  1

Et an 2  an1  an - TMBr½37 -

sikSaKNitviTüaedayxøÜnÉg

cMnYnkMupøic

1 i 3 eK)an zn1  an1  an  2 an1 1 i 3 zn 1  an  1  an 2 1 i 3 2 zn 1  ( an  1  an ) 2 1 i 3 1 i 3 1 i 3 zn 1  (an  1  an ) 2 2 1 i 3 z n 1  an 2

eK)an (zn ) CasIVútFrNImaRt éncMnYnkMupøicmanersug q  1  2i 3 . tamrUbmnþ zn  z 0  qn eday z0  a1  1  2i 3 a0  1 nig q  1  2i 3  cos 3  i sin 3  n  n n     eK)an zn (cos 3 i sin 3 ) cos 3 i sin 3 eday zn  an1  1  2i 3 an  (an1  a2n )  i 23 an eKTaj 23 an  sin n3 dUcenH an  23 sin n3 .

eroberogeday lwm plÁún

- TMBr½38 -

sikSaKNitviTüaedayxøÜnÉg

cMnYnkMupøic

16-Gnuvtþn_cMnYnkMupøickñúgFrNImaRt k> cm¶ayrvagBIrcMnuc y

B

A y1

O

A

x1

x2

x

eKmancMnYnkMupøicBIr z1  x1  i.y1 nig z 2  x2  i.y 2 . tag A nig B CacMnucrUbPaBén z1 nig z 2 kñúgbøg;kMupøic (xoy) . eK)an AB  (x2  x1 )2  (y 2  y1 )2 ehIy z 2  z1  (x2  x1 )  i(y 2  y1 ) eK)an | z 2  z1 | (x2  x1 )2  (y 2  y1 )2 dUcenH AB | z 2  z1 | .

eroberogeday lwm plÁún

- TMBr½39 -

sikSaKNitviTüaedayxøÜnÉg

cMnYnkMupøic

]TahrN_ 1 eK[BIrcMnuc A nig B manGahVikerogKña z1  1  2i nig z 2  2  6i cUrKNnacm¶ayrvagcMnuc A nig B eK)an AB | z 2  z1 | | ( 2  6i )  (1  2i ) |

| 3  4i | ( 3)2  42  5

dUcenH AB  5 . ]TahrN_2 eKmancMnYnkMupøic z1  2  a  i nig z 2  3  i(6  a) Edl a  IR kñúgbøg;kMupøic (xoy) eKtag A nig B CarUbPaBéncMnYnkMupøic z1 nig z 2 cUrkMNt;cMnYnBit a edIm,I[cm¶ayrvagcMnuc A nig B xøIbMput ? eK)an AB | z 2  z1 | | 3  i(6  a )  2  a  i |

| (1  a )  i(5  a ) | (1  a )2  (5  a )2  1  2a  a 2  25  10a  a 2  2a 2  12a  26  2(a  3)2  8

edIm,I[ AB xøIbMputluHRtaEt a  3 ehIy ABmin  eroberogeday lwm plÁún

82 2

.

- TMBr½40 -

sikSaKNitviTüaedayxøÜnÉg

cMnYnkMupøic

x> cMnucEckkñúgGgát;tampleFobmYy ³ kñúgbøg;kMupøic (xoy) eKtag A nig B CarUbPaBéncMnYnkMupøic z A nig zB yk P CacMnucmanGahVik z P CacMnucEckkñúgénGgát; AB tampleFob  Edl   0 . y

B P A

y

O

ebI P CacMnucEckkñúgénGgát; AB tampleFob  enaH  

 

 

AP   PB

 

eKman AP (z P  z A ) nig PB (z B  z P ) eK)an zp  z A  (z B  z P ) dUcenH z P  z A1 zB . eroberogeday lwm plÁún

- TMBr½41 -

sikSaKNitviTüaedayxøÜnÉg

cMnYnkMupøic

krNI   1 enaHcMnuc P CacMnuckNþalénGgát; [AB] dUcenHGahVikéncMnuc P kNþalGgát; AB KW z P  z A 2 zB . K> cMnucEckeRkAGgát;tampleFobmYy ³ kñúgbøg;kMupøic (xoy) eKtag A nig B CarUbPaBéncMnYnkMupøic z A nig zB yk Q CacMnucmanGahVik zQ CacMnucEckeRkAénGgát; AB tampleFob  Edl   0 . y

Q

B A

y

O

ebI Q CacMnucEckeRkAénGgát; AB tampleFob  enaH eKman

 

QA ( z A  z Q )

eroberogeday lwm plÁún

nig

 

 

QA   QB

 

QB ( z B  z Q )

- TMBr½42 -

sikSaKNitviTüaedayxøÜnÉg

cMnYnkMupøic

eK)an z A  zQ   (z B  zQ ) dUcenH zQ  z A1 zB . ]TahrN_ ³ eKmanBIrcMnuc A nig B CarUbPaBéncMnYnkMupøic z A  2  7i nig zB  1  i . 1 P CarUbPaBén z p CacMnucEckkñúgén AB tampleFob  p  3 CarUbPaBén zQ CacMnucEckeRkAén AB tampleFob  Q  23 cUrkMNt; z P nig zQ ? ebI P CarUbPaBén zp CacMnucEckkñúgén AB tampleFob  p  13 Q

eK)an

1 1 2  7 i   i 5 11 z A   PzB 3 3   i zP   1 1  P 4 2 1 3

.

ebI Q CarUbPaBén zQ CacMnucEckeRkAén AB tampleFob  Q  23 eK)an

z A   QzB zQ   1  Q

2 2  i 3 3  8  19i 2 1 3

2  7i 

dUcenH z P  54  112 i nig zQ  8  19i . eroberogeday lwm plÁún

- TMBr½43 -

sikSaKNitviTüaedayxøÜnÉg

cMnYnkMupøic

K> pleFobRCug nig mMuénRtIekaNkñúgbøg; kñúgbøg;kMupøic (xoy) eKmanbIcMnuc A , B , C CarUbPaBéncMnYnkMupøic z A , z B , zC . C

y

C' B A

B'

x

O

 

 

 

 

nig OC'  AC enaHeK)an OB' C' nig ABC CaRtIekaNb:unKña enaH BAC  B' OC' .

sg;viucTr½

OB'  AB

 

eKman OB' 

 

AB ( z B  z A )

eroberogeday lwm plÁún

nig

 

 

OC'  AC ( z C  z A )

- TMBr½44 -

sikSaKNitviTüaedayxøÜnÉg

cMnYnkMupøic

eKman B'OC'  XOC'XOB' b¤ BAC  arg(zC  z A )  arg(z B  z A )  arg zzC  zz A  

B

A



| z  z A | zC  z A ehIypleFobRCug AC .  C  AB | z  z | z  z B

A

B

A

dUcenHebI A(z A ) , B(zB ) , C(zC ) begáIt)anCaRtIekaN ABC enaH  z  zA  z  zA  . eK)an AC ni g  C BAC  arg C AB z  z z z  B

A



B

A



]TahrN_ ³ enAkñúgbøg;kMupøic ( XOY) eK[bIcMnuc A , B , C manGahVikerogKña ³ z A  2  i , z B  3  4i , z C   2  9i . cUrKNnapleFobRCug AC ni g BAC ? AB eKman zC  z A  2  9i  2  i  4  8i nig zB  z A  3  4i  2  i  1  3i eK)an zzC  zz A  1438i i  ((1438ii)()(1133ii))  2  2i B

A

2 2   i )  2 2 (cos  i sin ) 2 2 4 4  AC  2 2 nig BAC  . AB 4  2 2(

dUcenH

eroberogeday lwm plÁún

- TMBr½45 -

sikSaKNitviTüaedayxøÜnÉg

cMnYnkMupøic

X> sMNMucMNuckñúgbøg;kMupøic   kñúgbøg;kMupøicRbkbedaytRmuyGrtUNrm:al; (o, i , j ) eKtagcMnuc P(x; y ) CacMnucrUbPaBéncMnYnkMupøic z  x  i.y . sMNMuénrUbPaB P TaMgGs;EdlGacmaneTAtamlkçxNÐEdlRtUvbMeBjén z begáIt)anCasMNMuéncMnuc P kñúgbøg;kMupøic . ]TahrN_ 1 eK[cMnYnkMupøic z epÞógpÞat; (1  i )z  (1  i ) z  4 . P CarUbPaBéncMnYnkMupøic z kñúgbøg;kMupøic ( xoy ) . rksMNMucMnuc P ? tag z  x  i.y enaH z  x  i.y eKman (1  i )z  2(1  i ) z  4 eK)an ³ y

(1  i)(x  iy)  (1  i)(x  iy)  4 x  iy  ix  y  x  iy  ix  y  4 2x  2y  4  y  x  2

dUcenHsMNMucMnuc P KWCabnÞat; Edl mansmIkar ( ) : y  x  2

eroberogeday lwm plÁún

1

0

1

x

( ) : y  x  2

- TMBr½46 -

sikSaKNitviTüaedayxøÜnÉg

]TahrN_ 2 eK[cMnYnkMupøic z epÞógpÞat; | z  2  i | 3 . P CarUbPaBéncMnYnkMupøic z kñúgbøg;kMupøic ( xoy ) . rksMNMucMnuc P ? tag z  x  i.y CaGahVikéncMnuc P eKman | z  2  i | 3 eK)an | x  iy  2  i | 3

cMnYnkMupøic

y

1

0

1

x

| ( x  2)  i( y  1) | 3

( x  2)2  ( y  1)2  3

b¤ (x  2)2  (y  1)2  9 dUcenHsMMNMucMnuc P KWCargVg;p©it I(2 ,  1) nig kaM R  3 . ]TahrN_ 3 eK[cMnYnkMupøic w  zz  22  22ii ehIy P CacMnucrUbPaBén z kñúgbøg; (xoy) cUrkMNt;sMNMucMnuc P edIm,I[ w CacMnYnnimμitsuTæ ? edIm,I[ w CacMnYnnimμitsuTæluHRtaEt Re(w )  w 2 w  0 naM[ w   w eday w  zz  22  22ii nig w  zz  22  22ii eroberogeday lwm plÁún

- TMBr½47 -

sikSaKNitviTüaedayxøÜnÉg

cMnYnkMupøic

z  2  2i z  2  2i eK)an z  2  2i   z  2  2i ebI z  2  2i ( z  2  2i )( z  2  2i )  ( z  2  2i )( z  2  2i ) z z  ( 2  2i )z  ( 2  2i )z  8   z .z  ( 2  2i )z  ( 2  2i )z  8

b¤ | z |2  8 tag z  x  iy Edl (x, y )  (2 , 2) eK)an x2  y 2  8 . dUcenHsMNMucMnuc P CargVg;p©it O kaM R  2 2 elIkElgEtcMnuc A(2,2) . 2z z  16

y

x2  y 2  8 1

0

eroberogeday lwm plÁún

1

x

- TMBr½48 -

sikSaKNitviTüaedayxøÜnÉg

cMnYnkMupøic

]TahrN_ 4 eK[cMnYnkMupøic z ehIy P CacMnucrUbPaBén z kñúgbøg; (xoy) cUrkMNt;sMNMucMnuc P ebIeKdwgfa arg(z  2  2i )  4 . tag z  x  iy enaHeK)an z  2  2i  (x  2)  i(y  2) eday arg(z  2  2i )  4 enaH arg((x  2)  i(y  2))  4 eKTaj)an tan 4  yx  22  1 naM[ y  x  4 dUcenHsMNMucMnuc P CabnÞat; (d) : y  x  4 Edl xy  22  00 b¤ x  2 , y  2 . 

]TahrN_ 5 enAkñúgbøg;kMupøic (xoy) eK[cMnuc P(z ) , Q(iz ) , R(2  2i ) cUrkMNt;sMNMucMnuc P edIm,I[bIcMnuc P , Q , R rt;Rtg;Kña ?  

 

bIcMnuc P , Q , R rt;Rtg;KñaluHRtaEtRKb; t  IR : PQ  t QR b¤ zQ  z P  t(z R  zQ ) . tag z  x  iy enaH zQ  ix  y eK)an ix  y  x  iy  t(2  2i  ix  y ) (  x  y )  i( x  y )  ( 2  y )t  i( 2  x )t

eKTaj  x  y  (2  y )t nig (x  y )  (2  x)t eroberogeday lwm plÁún

- TMBr½49 -

sikSaKNitviTüaedayxøÜnÉg

cMnYnkMupøic

eK)an xxyy  22  yx b¤  2x  x2  2y  xy  2x  xy  2y  y 2 x 2  y 2  4x  4y  0 ( x  2)2  ( y  2)2  8

dUcenHsMNMucMnuc P CargVg;p©it I(2 , 2 ) mankaM R  2 2 . ]TahrN_ 6 enAkñúgbøg;kMupøic (xoy) eK[cMnuc P CarUbPaBén z EdlbMeBjlkçxNн z  2  2i 1  . cU r rksM N M u c M n u c P? zi 2 eK)an 2 | z  2  2i || z  i | Edl z  i b¤ 2 | z  2  2i |2 | z  i |2 tag z  x  i.y eK)an ³ 2 | x  iy  2  2i |2 | x  iy  i |2 2 | ( x  2)  i( y  2) |2 | x  i( y  1) |2 2[( x  2)2  ( y  2)2 ]  x 2  ( y  1)2 2x 2  8x  8  2y 2  8y  8  x 2  y 2  2y  1 x 2  y 2  8x  10y  15  0 ( x  4)2  ( y  5)2  26

dUcenHsMNMucMnuc P CargVg;p©it I(4 ,  5) nig kaM R  eroberogeday lwm plÁún

26

. - TMBr½50 -

sikSaKNitviTüaedayxøÜnÉg

cMnYnkMupøic

17-bMElgcMnuckñúgbøg;kMupøic k> bMElgkil kñúgbøg;kMupøic (xoy) eKmanBIrcMnuc M nig M' manGahVikerogKña z nig z'  M' CarUbPaBén M tambMElgkilénviucTr½ v ( z v ) eK)an ³ z'  z  z v . y M' M



v

x

O

ebI M' CarUbPaBén M tambMElgkilénviucTr½



 



v ( z v ) enaH MM'  v

 

eday MM'(z' z ) enaH z' z  z v b¤ z'  z  z v . eroberogeday lwm plÁún

- TMBr½51 -

sikSaKNitviTüaedayxøÜnÉg

cMnYnkMupøic

x> bMElgvilp©it  kñúgbøg;kMupøic (xoy) eKmanBIrcMnuc M nig M' manGahVikerogKña z nig z' M' CarUbPaBén M tambMElgvilp©it  ( z  ) nigmMu  eK)an ³ z' z   ( z  z  )(cos   i sin  ) . Y

y

M'



M

X



x O

ebI M' CarUbPaBén M tambMElgvilp©it (z  ) nigmMu  enaHeK)an M  M'  r nig MM'       Edl   XM' nig   XM . ehtuenH

z' z  r .ei  i  e i z  z  r .e

Edl     

dUcenH z' z   (z  z  )(cos   i sin  ) . eroberogeday lwm plÁún

- TMBr½52 -

sikSaKNitviTüaedayxøÜnÉg

cMnYnkMupøic

K> bMElgcaMgp©it  kñúgbøg;kMupøic (xoy) eKmanBIrcMnuc M nig M' manGahVikerogKña z nig z' M' CarUbPaBén M tambMElgcaMgp©it  ( z  ) pleFob  eK)an ³ z' z    ( z  z  ) . y M' M 

x O M'

CarUbPaBén M tambMElgcaMgp©it (z  ) pleFob  eK)an ³

 

 

 M'    M

dUcenH

eday

 

M'( z' z  ) ; M ( z  z  )

z' z    ( z  z  )

eroberogeday lwm plÁún

 

.

- TMBr½53 -

sikSaKNitviTüaedayxøÜnÉg

cMnYnkMupøic

18-)arIsg;énRbBn½æcMNuckñúgbøg;kMupøic eK[ Ak , k  1k n CaRbBn½æmYyman n cMNucb:ugeder Edlman n

GahVik z Ak . ebI  k   0 enaH)arIsg; G énRbBn½æmanGahVikmYy k 1 n

 ( k z A k )

kMNt;eday z G  k  1 n

  k 

.

k 1

tamniymn½y ebI G Ca)arIsg;énRbBnæ½ Ak , k  1k n enaHeK)an ³             GAk  OAk  OG    k GA k   O  k 1         n       k . OA k   k . OG   O  k 1   

eday

n

eK)an b¤ b¤

     OG .  (  k )     k . OA k   k 1 k 1   



n

n

n

n

k 1

k 1

z G .  ( k )   ( k z A k ) n

dUcenH

zG 

 ( k z A k )

k 1

n

  k 

.

k 1

eroberogeday lwm plÁún

- TMBr½54 -

sikSaKNitviTüaedayxøÜnÉg

cMnYnkMupøic

CMBUkTI2

kRmglMhat;eRCIserIsmandMeNaHRsay !> eKmancMnYnkMupøic z1  4  7i nig z 2  3  2i . k> cUrKNna z1  z 2 nig z1  z 2 . x> cUrKNna z1  z 2 nig zz1 . 2

@> eKman z1  1  2i nig z 2  3  i . cUrKNna U  z 12  z 2 2 nig V  z 13  z 2 3 ? #> cUrkMnt;cMnYnBit p nig q edIm,I[cMnYnkMupøic z  3  2i Cab¤srbs;smIkar z 2  ipz  q  0 . $> kMnt;BIrcMnYnBit x nig y ebIeKdwgfa ³ ( 3  i )(1  ix )  (1  3i )( 3  2iy ) 

2(7  9i ) 1 i

.

%> cUrKNnab¤skaerén z  40  42i . ^> eK[cMnYnkMupøic a  2  3i ; b  3  i nig c  1  4i k-cUrsresr a 3  b 3  c 3 nig a  b  c CaTRmg;BICKNit . x-cUrepÞógpÞat;faeKGackMnt;cMnYnBit k edIm,I[ a 3  b 3  c 3  k . a b c eroberogeday lwm plÁún

- TMBr½55 -

sikSaKNitviTüaedayxøÜnÉg

cMnYnkMupøic

&> eK[cMnYnkMupøic a  3  i ; u  (x  1)  i.(y  2) nig b  2  16i Edl x nig y CaBIrcMnYnBit . cUrkMnt;témø x nig y edIm,I[ au  b  0 . *> eK[cMnYnkMupøic Z  log 3 ( x 2 y )  i log 2 x  log 2 y 

i nig W  13 Edl x  IR * ; y  IR * . 1  2i k-cUrsresr W CaTRmg;BICKNit . x-kMnt; x nig y edIm,I[ Z  W . (>eK[smIkar (E): z 2  az  b  0 Edl a ;b  IR cUrkMnt;témø a nig b edIm,I[cMnYnkMupøic z1  2  i 3 Cab¤srbs; smIkar (E) rYcTajrkb¤s z 2 mYYyeTotrbs;smIkar . etIGñkBinitüeXIjdUcemþccMeBaHcMnYnkMupøic z1 nig z 2 ? !0> eK[cMnYnkMupøic z Edlman z CacMnYnkMupøicqøas;rbs;va . edaHRsaysmIkar log 5 | z |  z 7i z  3  i !!> eK[cMnYnkMupøicBIr ³ z  3x  i ( 2x  y ) nig W  1  y  i [ 1  2 log5 ( x  3 ) ] Edl x nig y CacMnYnBit . kMnt; x nig y edIm,I[ W  z .

eroberogeday lwm plÁún

- TMBr½56 -

sikSaKNitviTüaedayxøÜnÉg

cMnYnkMupøic

!@> edaHRsaysmIkar 2z  | z |  917ii Edl z CacMnYnkMupøic . !#> eK[cMnYnkMupøicBIr Z1 nig Z 2 Edl Z 2  0 . cUrRsaybBa¢ak;fa ZZ1  || ZZ1 || . 2

2

!$> eK[cMnYnkMupøicBIr Z1 nig Z 2 . cUrRsaybBa¢ak;fa | Z1  Z 2 |  | Z1 | | Z 2 | . !%> eK[cMnYnkMupøicBIr Z1 nig Z 2 . k> cUrRsaybBa¢ak;fa | Z1  Z 2 |  | Z1 |  | Z 2 | x> Taj[)anfa (a  c) 2  (b  d) 2  a 2  b 2  cMeBaHRKb; a , b , c , d CacMnYnBit . !^> eK[cMnYnkMupøic ³ z1  6 2i. 2 nig z 2  1  i k>cUrsresr

z 1 , z 2 nig Z 

z1 z2

c2  d2

CaragRtIekaNmaRt.

x>cUrsresr Z  zz1 CaragBiCKNit. 2

K>Taj[)anfa cos 12 

eroberogeday lwm plÁún

6 2 4

nig sin 12 

6 2 . 4

- TMBr½57 -

sikSaKNitviTüaedayxøÜnÉg

!&> eK[cMnYnkMupøic z  2  2  i. 2  k-cUrsresr z 2 CaTMrg;BiCKNit . x-cUrsresr z 2 nig z CaTMrg;RtIekaNmaRt . K-TajrktMélR)akdén cos 8 nig sin 8 .

cMnYnkMupøic 2

.

!*> eK[cMnYnkMupøic Z  cos 45  i. sin 45

cUrRsaybBa¢ak;fa (1  Z) 3  8 cos 3 25 (cos 65  i. sin 65 ) . !(> eK[cMnYnkMupøic Z  4 2(1  i ) k-cUrsresr Z CaTRmg;RtIekaNmaRt . x-KNnab¤sTI# én Z . @0> eKeGay f z   z 3  2 3  i .z 2  41  i 3 .z  8i k>cUrbgðajfa z ¢ f z   z  2i z 2  2 3z  4 x>edaHRsaysmIkar f z   0 kñúgsMNMukMupøic . @!> eK[cMnYnkMupøic z  cos 47  i. sin 47 . cUrsresr (1  z )4 CaragRtIekaNmaRt .

eroberogeday lwm plÁún

- TMBr½58 -

sikSaKNitviTüaedayxøÜnÉg

@@> eK[cMnYnkMupøic k-cUrsresr z 1

z1  1  i

,z2

cMnYnkMupøic

nig

6  i. 2 z2  2

nig zz1 CaTMrg;RtIekaNmaRt . 2

x-cUrsresr zz1 CaTMrg;BiCKNit . 2

K-eRbIlTæplxagelIcUrTajrktMélR)akdén cos 12 nig sin 12 . @#> eK[sIVúténcMnYnkMupøic (Z n ) kMnt;eday  1 i 3 Z   0 2  1 Z  n  1  2  Z n  | Z n |  ; n  IN ¬ | Z n | Cam:UDulén Z n ¦.

snμtfa Z n   n (cos n  i. sin n ) , n  IN Edl  n  0 ,  n ; n  IR . k-cUrrkTMnak;TMngrvag n nig  n1 ehIy n nig  n1 . x-rkRbePTénsIVút (n ) rYcKNna n CaGnuKmn_én n . K-cUrbgðajfa  n   0 cos  0 cos 21 cos 22 .... cos  n2 1 rYcbBa¢ak; n GnuKmn_én n . eroberogeday lwm plÁún

- TMBr½59 -

sikSaKNitviTüaedayxøÜnÉg

cMnYnkMupøic

@$> eK[cMnYnkMupøic z  x  i.y Edl x nig y CaBIrcMnYnBit . cUrkMnt;témø x nig y ebIeKdwgfa ³ 10 ( 3  2i ) z  (1  3i ) z  ¬ z CacMnYnkMupøicqøas;én z ¦ . 2i @%> eK[ z ; z CacMnYnkMupøicEdl | z || z | r  0 . 1

2

1

2

2

2

 z1  z 2   z1  z 2  1  2    2   2 r  r  z 1z 2   r  z 1z 2 

bgðajfa @^>eKyk z

1

; z 2 ; ....; z n

CacMnYnkMupøicEdlepÞógpÞat;TMnak;TMng

(k  1)z k  1  i(n  k )z k  0 ; k  0 , 1 , 2 , ... , n  1

k-kMnt; z ebIeKdwgfa z  z  z  ....  z  2 x-cMeBaHtémø z Edl)ankMnt;xagelIcUrbgðajfa ³ 0

0

1

2

n

n

0

( 3n  1)n | z 0 |  | z1 |  | z 2 | .... | z n |  n! 2

@&> eK[ z

2

1

2

2

CacMnYnkMupøicedaydwgfa ³ z z z z 0 .

; z2 ; z3

z 1  z 2  z 3  z 1z 2

cUrbgðajfa | z @*> cUrbgðajfa

1

2 3

3 1

|| z 2 || z 3 | 6z  i  1 2  3iz

eroberogeday lwm plÁún

luHRtaEt

|z|

1 3

- TMBr½60 -

sikSaKNitviTüaedayxøÜnÉg

@(> eK[ z

1

cMnYnkMupøic

CacMnYnkMupøicEdlmanm:UDúlesμI 1 . 1    )     ( )  . z 

; z 2 ; z 3 ; ....; z n

 n Z    (zk  k 1

eKtag   cUrbgðajfa 0  Z  n #0> eK[cMnYnkMupøic z Edl | z | 1 . cUrbgðajfa ³ 2  |1 z |  |1 z |  4 . n

k 1

k

2

2

#!> eK[cMnYnkMupøic Z  (cos x  1 )  i.(sin x  1 ) cos x sin x Edl x CacMnYnBit. cUrkMnt´rkm¨UDulGb,brmaéncMnYnkMupøicen¼ ? #@> eK[ A   13  i    13  i  , n  IN . 2

2

2

n



cUrbgðajfa

A  i.

( 3)

n





. sin

n 3

2 n 1 n

2



cMeBaHRKb; n  IN .

 

##> kñúgbøg;kMupøic (O , i , j ) eK[bYncMnuc A , B , C , D

EdlmanGahVikerogKña nig Z  2  3i . cUrRsayfactuekaN ABCD carikkñúgrgVg;mYyEdleKnwgbBa¢ak;p©it nig kaMrbs;va . Z A  1  6i , Z B  4  5i , Z C  5  4i

eroberogeday lwm plÁún

D

- TMBr½61 -

sikSaKNitviTüaedayxøÜnÉg

cMnYnkMupøic

#$> eK[ z nig z CaBIrcMnYnkMupøic . 1

2

cUrbgðajfa | 1  z z 1

| z 1  1 || z 2  1 | 2 |  | z1  z 2 |  2

2

#%> eK[ z nig z CacMnYnkMupøicBIr . cUrRsayfa | z  z |  | z  z |  2| z |  | z |  #^>eK[ z nig z CacMnYnkMupøicBIrEdl | z |  | z |  1 nig z .z  1 . cUrRsayfa 1z z zz CacMnYnBitmYy . #&> eK[BhuFa P(x)  (x sin a  cos a) Edl n  IN * cUrrksMNl´énviFIEckrvag P(x) nwg x  1 . #*> eK[cMnYnkMupøic z  2  cos   i 2  sin  1

2

2

1

1

1

2

2

1

2

2

1

2

2

2

1

2

2

1

2

1 2

n

2

Edl   IR .   kñúgbøg;kMpøic (o , i , j ) eKehA M CacMnucrUbPaBén z . cUrkMNt;témøtUcbMput nig FMbMputén r  OM ? #(> eK[cMnYnkMupøic z1 , z 2 , z 3 ehIyepÞógpÞat;TMnak;TMng ³ | z 1 || z 2 || z 3 | 1 nig

z 32 z12 z 22   1 0 z 2 z 3 z 1z 3 z 1z 2

cUrRsayfa | z1  z 2  z 3 |  { 1 , 2 } . eroberogeday lwm plÁún

- TMBr½62 -

sikSaKNitviTüaedayxøÜnÉg

$0> eK[cMnYnkMupøic z1 nig

cMnYnkMupøic z2

Edl | z1 || z 2 | 1

cUrRsayfa | z1  1 |  | z 2  1 |  | z1z 2  1 | 2 $!> eK[sIVútcMnYnBit (an ) kMNt;eday ³

a 1  1 , a 2  1  a n  2  a n 1  a n , n  1 , 2 , 3 , ... 1 i 3 eKtagsIVútcMnYnkMupøic z n  an1  an . 2 k> cUrRsayfa z n1  1  i 3 z n cMeBaHRKb; n  1 . 2 x> cUrdak; 1  i 3 CaTRmg;RtIekaNmaRtrYcTajrk z n CaGnuKmn_én n 2 K> TajrktYTUeTAénsIVút an . etI (an ) CasIVútxYbb¤eT ?

.

$@> eK[sIVúténcMnYnkMupøic ( z n ) kMNt;eday ³ 2 3i nig z n1  3  i z n  2  3  i 2 2 2 Edl n  1 , 2 , 3 , ... . z1 

k> tag w n  z n  1 . bgðajfa (w n ) CasIVútFrNImaRténcMnYnkMupøic rYcKNna w n CaGnuKmn_én n edaysresrlTæplCaTRmg;RtIekaNmaRt . eroberogeday lwm plÁún

- TMBr½63 -

sikSaKNitviTüaedayxøÜnÉg

cMnYnkMupøic

x> Tajbgðajfa z n  2 cos n (cos n  i sin n ) . 12 12 12 $#> eK[sIVúténcMnYnBit (u n ) nig ( v n ) kMNt;eday ³ un  vn   u  n 1 2 nig  un  vn v   n 1 2 k> eKBinitüsIVúténcMnYnkMupøic z n  un  i.v n

 u1   v   1

2 2 2 2

Edl n  1

. cUrRsayfa (z n ) CasIVútFrNImaRténcMnYnkMupøic rYcKNna z n CaGnuKmn_én n edaysresrlTæplCaTRmg;RtIekaNmaRt . x> sMEdg un nig v n CaGnuKmn_én n . $$> eK[sIVúténcMnYnBit (un ) nig ( v n ) kMNt;eday ³ u 0  1  v0  3

u n  1  u n 2  v n 2 nig   v n 1  2u n v n k> eKBinitüsIVúténcMnYnkMupøic z n  un  i.v n .

Edl n  0 2n

cUrRsayfa z n1  z n rYcTajfa z n  z 0 . x> sMEdg un nig v n CaGnuKmn_én n . 2

eroberogeday lwm plÁún

- TMBr½64 -

sikSaKNitviTüaedayxøÜnÉg

cMnYnkMupøic

lMhat;TI1 eKmancMnYnkMupøic z1  4  7i nig z 2  3  2i . k> cUrKNna z1  z 2 nig z1  z 2 . x> cUrKNna z1  z 2 nig zz1 . 2

dMeNaHRsay k> KNna z1  z 2 nig z1  z 2 eyIg)an z1  z 2  (4  7i )  (3  2i )  4  7i  3  2i  7  9i nig z1  z 2  (4  7i )  (3  2i )  4  7i  3  2i  1  5i dUcenH z1  z 2  7  9i nig z1  z 2  1  5i . x> KNna z1  z 2 nig zz1

2

eyIg)an z1  z 2  (4  7i )(3  2i )  12  8i  21i  14  2  29i nig z1  4  7i  (4  7i)(3  2i)  12  8i  21i  14  26  13i  2  i z2

dUcenH

3  2i

( 3  2i )( 3  2i )

z1  z 2  2  29i

eroberogeday lwm plÁún

34

nig

z1  2i z2

13

.

- TMBr½65 -

sikSaKNitviTüaedayxøÜnÉg

cMnYnkMupøic

lMhat;TI2 eKman z1  1  2i nig z 2  3  i . cUrKNna U  z 12  z 2 2 nig V  z 13  z 2 3 ? dMeNaHRsay KNna U  z 12  z 2 2 nig V  z 13  z 2 3 eyIg)an ³

U  1  2i 2  3  i 2  1  4i  4  9  6i  1  5  10i

V  1  2i 3  3  i 3  1  6i  12  8i  27  27i  9  i  7  24i

dUcenH

U  5  10i

eroberogeday lwm plÁún

nig

V  7  24i

.

- TMBr½66 -

sikSaKNitviTüaedayxøÜnÉg

cMnYnkMupøic

lMhat;TI3 cUrkMnt;cMnYnBit p nig q edIm,I[cMnYnkMupøic z  3  2i Cab¤srbs;smIkar z 2  ipz  q  0 . dMeNaHRsay kMnt;cMnYnBit p nig q ebI z  3  2i Cab¤srbs;smIkar z 2  ipz  q  0 enaHvaRtUvepÞógpÞat;smIkar eK)an 3  2i 2  ip3  2i   q  0

9  12i  4  3ip  2p  q  0 5  2p  q   i.12  3p   0 eKTaj)an 5  2p  q  0 naM[ q  2p  5  13 12  3p  0 p   4

dUcenH

p  4 , q  13

eroberogeday lwm plÁún

.

- TMBr½67 -

sikSaKNitviTüaedayxøÜnÉg

lMhat;TI4 kMnt;BIrcMnYnBit x nig y ebIeKdwgfa ³ ( 3  i )(1  ix )  (1  3i )( 3  2iy ) 

cMnYnkMupøic

2(7  9i ) 1 i

.

dMeNaHRsay kMnt;BIrcMnYnBit x nig y eKman (3  i )(1  ix)  (1  3i )(3  2iy )  2(17  i9i )

eK)an 3  3ix  i  x  3  2iy  9i  6y  2(7  9i )(1  i )

11  x  6y  6  3ix  2iy  8i   2(7  7i  9i  9) 2  x  6y  6  i.3x  2y  8  16  2i   x  6y  6  16 eKTaj   3x  2y  8  2 b¤  x  6y  10 naM[ x  2 , y  2 .  3x  2y  10

dUcenH

x2 , y2

eroberogeday lwm plÁún

.

- TMBr½68 -

sikSaKNitviTüaedayxøÜnÉg

cMnYnkMupøic

lMhat;TI5 cUrKNnab¤skaerén z  40  42i . dMeNaHRsay tag W  x  i.y , x; y  IR Cab¤skaerén z  40  42i eK)an W 2  z eday W 2  (x  i.y ) 2  (x 2  y 2 )  2ixy  x 2  y 2  40 2 2 naM[ (x  y )  2ixy  40  42.i eKTaj   2xy  42

eday x  y   (x2  y 2 )2  4x2y 2  402  422  3364 eKTaj x 2  y 2  3364  58 . 2

2 2

 x 2  y 2  58 1 eK)anRbBnæ½smIkar  2 2  x  y  40 2  bUksmIkar ¬!¦ nig ¬@¦ eK)an 2x 2  98 naM[ x  7

dksmIkar¬!¦ nig ¬@¦ eK)an 2y 2  18 naM[ y  3 eday 2xy  42  0 naM[ x nig y mansBaØadUcKña naM[eKTaj)anKUcemøIy ³ x  7 , y  3 nig x  7 , y  3 . dUcenH W1  7  3i nig W2  7  3i .

eroberogeday lwm plÁún

- TMBr½69 -

sikSaKNitviTüaedayxøÜnÉg

cMnYnkMupøic

lMhat;TI6 eK[cMnYnkMupøic a  2  3i ; b  3  i nig c  1  4i k-cUrsresr a 3  b 3  c 3 nig a  b  c CaTRmg;BICKNit . x-cUrepÞógpÞat;faeKGackMnt;cMnYnBit k edIm,I[ a 3  b 3  c 3  k . a b c . dMeNaHRsay k¿sresr a 3  b 3  c 3 nig a  b  c CaTRmg´BICKNit ½ eyIgman a 3  (2  3i ) 3  8  36i  54  27i  46  9i b 3  ( 3  i ) 3  27  27i  9  i  18  26i c 3  (1  4i ) 3  1  12i  48  64i  47  52i

eyIgán a 3  b 3  c3  46  9i  18  26i  47  52i  111  87i

.

dUcen¼

a 3  b 3  c 3  111  87i

eyIgman

a  b  c  ( 2  3i )( 3  i )(1  4i )  ( 6  2i  9i  3)(1  4i )

 ( 9  7i )(1  4i )  9  36i  7i  28  37  29i

dUcen¼

a  b  c  37  29i

eroberogeday lwm plÁún

. - TMBr½70 -

sikSaKNitviTüaedayxøÜnÉg

cMnYnkMupøic

x¿kMnt´cMnYnBit k eyIgman a 3  b 3  c 3  k .a b c eKTaj dUcen¼

a 3  b 3  c 3  111  87i  k 3 a.b.c  37  29i k3 .

eroberogeday lwm plÁún

- TMBr½71 -

sikSaKNitviTüaedayxøÜnÉg

cMnYnkMupøic

lMhat;TI7 eK[cMnYnkMupøic a  3  i ; u  (x  1)  i.(y  2) nig b  2  16i Edl x nig y CaBIrcMnYnBit . cUrkMnt;témø x nig y edIm,I[ au  b  0 . dMeNaHRsay kMnt;témøén x nig y eyIg)an au  b  0 b a eday a  3  i ; u  (x  1)  i.(y  2) nig b  2  16i eK)an (x  1)  i.(y  2)   2316i i 2(1  8i )( 3  i ) ( x  1)  i( y  2)   32  i 2 2( 3  i  24i  8) ( x  1)  i( y  2)   10 ( x  1)  i( y  2)  1  5i eKTaj)an  xy  12  51 naM[ x  2 ; y  3  u

dUcenH

x2 ; y3

eroberogeday lwm plÁún

. - TMBr½72 -

sikSaKNitviTüaedayxøÜnÉg

cMnYnkMupøic

lMhat;TI8 i eK[cMnYnkMupøic Z  log 3 ( x 2 y )  i log 2 x  log 2 y  nig W  13 1  2i Edl x  IR* ; y  IR * . k-cUrsresr W CaTRmg;BICKNit . x-kMnt; x nig y edIm,I[ Z  W . dMeNaHRsay k-sresr W CaTRmg;BICKNit  i (12  i )(1  2i ) 12  24i  i  2 eyIg)an W  12    2  5i 1  2i 1 4 5 dUcenH

W  2  5i

.

x-kMnt; x nig y edIm,I[ Z  W

xy  log ( )2  2 naM[ eyIg)an Z  W smmUl  3 log 2 x  log 2 y  5 eKTaj x ; y Cab¤sssmIkar u 2  12u  32  0 u 1  6  2  4 eday '  36  32  4 eKTajb¤s u  6  2  8  2 dUcenH x  4 ; y  8 b¤ x  8 ; y  4 .

eroberogeday lwm plÁún

 x  y  12   x.y  32

- TMBr½73 -

sikSaKNitviTüaedayxøÜnÉg

cMnYnkMupøic

lMhat;TI9 eK[smIkar (E): z 2  az  b  0 Edl a ;b  IR cUrkMnt;témø a nig b edIm,I[cMnYnkMupøic z1  2  i 3 Cab¤srbs; smIkar (E) rYcTajrkb¤s z 2 mYYyeTotrbs;smIkar . etIGñkBinitüeXIjdUcemþccMeBaHcMnYnkMupøic z1 nig z 2 ? dMeNaHRsay kMnt;témø a nig b edIm,I[cMnYnkMupøic 2  i 3 Cab¤srbs;smIkarluHRtaEtvaepÞógpÞat;smIkar eK)an (2  i 3 )2  a(2  i 3 )  b  0 4  4i 3  3  2a  ai 3  b  0 (1  2a  b )  i(4 3  a 4 3  a 3  0 eKTaj)an  b¤  1  2a  b  0

dUcenH

a  4 ; b  7

3)  0 a   4  b  7

.

KNnab¤smYYyeTotrbs;smIkar ³ ebI z1 ; z 2 Cab¤srbs;smIkarenaHtamRTwsþIbTEvüteK)an ³ z 1  z 2   a  4 eKTaj z 2  4  ( 2  i 3 )  2  i 3 dUcenH z 2  2  i 3 . eyIgBinitüeXIjfacMnYnkMupøic z1 nig z 2 qøas;Kña . eroberogeday lwm plÁún

- TMBr½74 -

sikSaKNitviTüaedayxøÜnÉg

cMnYnkMupøic

lMhat;TI10 eK[cMnYnkMupøic z Edlman z CacMnYnkMupøicqøas;rbs;va . edaHRsaysmIkar log 5 | z |  z 7i z  3  i dMeNaHRsay edaHRsaysmIkar log 5 | z |  z 7 i z  3  i tag z  x  i.y naM[ z  x  iy Edl x ; y  IR smIkarGacsresr ³

( x  iy )  i( x  iy ) 3i 7 x  iy  ix  y log 5 x 2  y 2  3i 7 xy xy (  log 5 x 2  y 2 )  i 3i 7 7 x  y  x  y  7  7  1 eKTaj)an  x  y b¤  2 2  1 log x y   3 2 2   5    log x y 3 5  7 x  y  7 b¤  2 2 eKTajcemøIy x  3 , y  4 b¤ x  4 , y  3 .  x  y  25 dUcenH z1  3  4i ; z 2  4  3i CacemøIyrbs;smIkar . log 5 | x  iy | 

eroberogeday lwm plÁún

- TMBr½75 -

sikSaKNitviTüaedayxøÜnÉg

cMnYnkMupøic

lMhat;TI11 eK[cMnYnkMupøicBIr ³

nig W  1  y  i [ 1  2 log5 ( x  3) ] Edl x nig y CacMnYnBit . kMnt; x nig y edIm,I[ W  z . dMeNaHRsay kMnt; x nig y Re( W )  Re( z ) edIm,I[ W  z luHRtaEt Im( W )  Im( z ) z  3x  i ( 2x  y )

 (1)   1  y  3x eKTaj)an  log 5 ( x  3)  2 x  y ( 2) 1  2 tam (1) eKTaj y  3x  1 (3) ykCYskñúgsmIkar (2)

eK)an

1  2 log5 ( x  3 )  2x  3x  1

2 log5 ( x 3)  x

lkçx½NÐ x  3  0 b¤ x  3 tag t  log 5 (x  3) naM[ x  3  5t b¤ x  5t  3 eK)an smIkar 2t  5t  3 b¤ 5t  2t  3 edayGgÁxageqVgénsmIkarCaGnuKmn_ekIn nig GgÁxagsþaMCaGnuKmn_efr enaHeyIg)ansmIkarmanb¤sEtmYyKt;KW t  1 . cMeBaH t  1 eK)an x  5  3  2 nig y  3(2)  1  7 dUcenH x  2 ; y  7 . eroberogeday lwm plÁún

- TMBr½76 -

sikSaKNitviTüaedayxøÜnÉg

cMnYnkMupøic

lMhat;TI12 edaHRsaysmIkar 2z  | z |  917ii Edl z CacMnYnkMupøic . dMeNaHRsay edaHRsaysmIkar 9  7i 2z  | z |  tag z  x  i .y , x; y  IR 1 i eK)an 2(x  iy ) | x  iy | ((917ii)()(11ii)) 2x  2iy  x 2  y 2 

9  9i  7 i  7 2

( 2x  x 2  y 2 )  2iy  1  8i

 2x  x 2  y 2  1 (1) eKTaj   2y  8 ( 2) tam (2) eKTaj y  4 ykeTACYskñúg (1) eK)an 2x  x 2  16  1 2x  1  x 2  16

, x

1 2

( 2x  1) 2  x 2  16 4x 2  4x  1  x 2  16 3x 2  4x  15  0

;  '  4  45  7 2

eroberogeday lwm plÁún

- TMBr½77 -

sikSaKNitviTüaedayxøÜnÉg

eKTajb¤s x1  2 3 7  3 eK)an x  3 ; y  4 . dUcenH

z  3  4i

cMnYnkMupøic ; x2 

27 5 1   3 3 2

¬minyk¦

CacemøIyrbs;smIkar .

eroberogeday lwm plÁún

- TMBr½78 -

sikSaKNitviTüaedayxøÜnÉg

cMnYnkMupøic

lMhat;TI13 eK[cMnYnkMupøicBIr Z1 nig Z 2 Edl Z 2  0 . cUrRsaybBa¢ak;fa ZZ1  || ZZ1 || . 2

dMeNaHRsay RsaybBa¢ak;fa

2

Z1 |Z |  1 Z2 | Z2 |

eyIgtag Z1  a  i.b nig Z 2  c  i . d Edl a , b , c , d CacMnYnBit . Z 1 a  i .b (a  i .b )(c  i .d ) ac  i .ad  i .bc  i 2 .bd eyIg)an    2 2 2 Z2

eK)an naM[

c  i .d

(c  i .d )(c  i .d )

c  i .d

Z 1 (ac  bd )  i(bc  ad ) ac  bd bc  ad   i .  Z2 c2  d2 c2  d2 c2  d2 2

Z1  ac  bd   bc  ad    2    2  2 2 Z2 c d  c d     

2

(ac  bd ) 2  (bc  ad ) 2 c2  d2 a 2c 2  2abcd  b 2d 2  b 2 c 2  2abcd  a 2d 2 c2  d2 (a 2 c 2  a 2 d 2 )  ( b 2 c 2  b 2 d 2 ) c2  d2 a 2 ( c 2  d 2 )  b 2 (c 2  d 2 )

eroberogeday lwm plÁún

c d 2

2



(c 2  d 2 )(a 2  b 2 ) c2  d2

- TMBr½79 -

sikSaKNitviTüaedayxøÜnÉg

eKTaj

Z1  Z2

cMnYnkMupøic

(a 2  b 2 )(c2  d 2 ) (c  d ) 2

2 2



a2  b2 c2  d 2

 | Z |  a2  b2 edayeKman  1  | Z 2 |  c 2  d 2 Z1 |Z |  1 . dUecñH Z2 | Z2 |

eroberogeday lwm plÁún

- TMBr½80 -

sikSaKNitviTüaedayxøÜnÉg

cMnYnkMupøic

lMhat;TI14 eK[cMnYnkMupøicBIr Z1 nig Z 2 . cUrRsaybBa¢ak;fa | Z1  Z 2 |  | Z1 | | Z 2 | . dMeNaHRsay RsaybBa¢ak;fa | Z1  Z 2 |  | Z1 | | Z 2 | eyIgtag Z1  a  i.b nig Z 2  c  i . d Edl a , b , c , d CacMnYnBit . eyIgman Z1  Z 2  (a  i.b)(c  i.d)  ac  i.ad  i.bc  i 2 .bd naM[ Z1  Z 2  (ac  bd)  i.(ad  bc) eK)an | Z1  Z 2 |  (ac  bd) 2  (ad  bc) 2  a 2 c 2  2abcd  b 2 d 2  a 2 d 2  2abcd  b 2 c 2  (a 2 c 2  b 2 c 2 )  ( a 2 d 2  b 2 d 2 )  c 2 (a 2  b 2 )  d 2 (a 2  b 2 )  (a 2  b 2 )(c 2  d 2 )

eKTaj

| Z1  Z 2 |  a 2  b 2  c 2  d 2

edayeKman dUcenH

 | Z |  a2  b2  1   | Z 2 |  c 2  d 2 | Z 1  Z 2 |  | Z 1 | | Z 2 |

eroberogeday lwm plÁún

.

- TMBr½81 -

sikSaKNitviTüaedayxøÜnÉg

cMnYnkMupøic

lMhat;TI15 eK[cMnYnkMupøicBIr Z1 nig Z 2 . k> cUrRsaybBa¢ak;fa | Z1  Z 2 |  | Z1 |  | Z 2 | x> Taj[)anfa (a  c) 2  (b  d) 2  a 2  b 2  c 2  d 2 cMeBaHRKb; a , b , c , d CacMnYnBit . dMeNaHRsay k> RsaybBa¢ak;fa | Z1  Z 2 |  | Z1 |  | Z 2 |   kñúgbøg;kMupøic ( XOY) eyIgeRCIserIsviucTr½BIr U nig V manGahVikerogKña   Z 1 nig Z 2 naM[viucTr½ U  V manGahVik Z 1  Z 2 . tamlkçN³RCugrbs;RtIekaNeK)an     U V | U  V |  | U |  | V | eday ³ 









U

| U |  | Z1 | , | V |  | Z 2 | 





| U  V || Z1  Z 2 |

dUcenH

V

| Z1  Z 2 |  | Z1 |  | Z 2 |

eroberogeday lwm plÁún

- TMBr½82 -

sikSaKNitviTüaedayxøÜnÉg

cMnYnkMupøic

x> Taj[)anfa (a  c) 2  (b  d) 2  a 2  b 2  c 2  d 2 eyIgtag Z1  a  i.b nig Z 2  c  i . d Edl a , b , c , d CacMnYnBit . man Z1  Z 2  (a  c)  i.(b  d) naM[ | Z1  Z 2 | (a  c) 2  (b  d) 2 ehIy | Z1 | a 2  b 2 , | Z 2 | c 2  d 2 . tamsRmayxagelIeKman | Z1  Z 2 |  | Z1 |  | Z 2 | dUcenH

(a  c ) 2  ( b  d ) 2  a 2  b 2  c 2  d 2

eroberogeday lwm plÁún

.

- TMBr½83 -

sikSaKNitviTüaedayxøÜnÉg

cMnYnkMupøic

lMhat;TI16 eK[cMnYnkMupøic ³ z1 

6  i. 2 ni g z2  1  i 2 k>cUrsresr z 1 , z 2 nig Z  zz1 CaragRtIekaNmaRt. 2 x>cUrsresr Z  zz1 CaragBiCKNit. 2 K>Taj[)anfa cos 12  6 4 2 nig sin 12  6 4 2 .

dMeNaHRsay z1 k>sresr z 1 , z 2 nig Z  z CaragRtIekaNmaRt³ 2

 3 6i 2 1     2   i .   2  cos  i . sin  2 2 6 6   2     z 1  2 cos(  )  i . sin(  ) . 6 6  

eKman z1  dUcenH

 2 2      2  cos  i . sin  2   i. 2  4 4   2     z 2  2 cos(  )  i . sin(  ) . 4 4  

eKman z 2  1  i  dUcenH

eroberogeday lwm plÁún

- TMBr½84 -

sikSaKNitviTüaedayxøÜnÉg

cMnYnkMupøic

2      cos( ) i . sin(      )  6 4 6 4  2 2   Z  cos  i . sin . 12 12

eKman Z  zz1  dUcenH

z1 x> sresr z CaragBiCKNit 2 eK)an Z  26(1i i )2  ( 26(1i i )(21)(1 i ) i )  Z

dUcenH

Z

6 2 6 2  i. 4 4

K> TajeGay)anfa cos 12  tamsRmayxagelIeKman ³ Z  cos

   i . sin 12 12

(1)

6 2 4

6i 6i 2 2 4

. nig sin 12 

nig Z 

6 2 4

6 2 6 2  i. 4 4

( 2)

edaypÞwm ¬!¦ nig ¬@¦ eKTaj)an ³ cos

  12

6 2 4

eroberogeday lwm plÁún

nig

sin

  12

6 2 4

.

- TMBr½85 -

sikSaKNitviTüaedayxøÜnÉg

cMnYnkMupøic

lMhat;TI17 eK[cMnYnkMupøic z  2  2  i. 2  2 . k-cUrsresr z 2 CaTMrg;BiCKNit . x-cUrsresr z 2 nig z CaTMrg;RtIekaNmaRt . K-TajrktMélR)akdén cos 8 nig sin 8 . dMeNaHRsay k-sresr z 2 CaTMrg;BiCKNit eyIg)an z 2  ( 2  2  i. 2  2 )2 



2 2

2  2i

2 2



 

2  2  i. 2  2

2

 2  2  2i . 4  2  2  2  2 2  2 2 .i

dUcenH

z 2  2 2  2 2 .i

x-sresr z 2 nig z CaTMrg;RtIekaNmaRt eKman z 2  2 2  2 2 .i dUcenH

 2   2     4 cos  i . sin   4  i. 2  4 4   2       z 2  4 cos  i . sin  nig z  2 cos  i . sin  4 4 8 8  

eroberogeday lwm plÁún

- TMBr½86 -

sikSaKNitviTüaedayxøÜnÉg

cMnYnkMupøic

K-TajrktMélR)akdén cos 8 nig sin 8

tamsMrayxagelIeKman z  2 cos 8  i. sin 8    eday z  2  2  i. 2  2 eKTaj 2 cos 8  i. sin 8   2  2  i. 2  

dUcenH

cos



 2 2  8 2

eroberogeday lwm plÁún

nig

sin

 2 2  8 8

2

.

- TMBr½87 -

sikSaKNitviTüaedayxøÜnÉg

lMhat;TI18 eK[cMnYnkMupøic Z  cos 45  i. sin 45 cUrRsaybBa¢ak;fa dMeNaHRsay RsaybBa¢ak;fa ³

(1  Z ) 3  8 cos 3

cMnYnkMupøic

2 6 6 (cos  i . sin ) 5 5 5

.

2 6 6 (cos  i . sin ) 5 5 5 eyIgman 1  Z  1  cos 45  i. sin 45 tamrUbmnþ 1  cos   2 cos 2 2 nig sin   2 sin 2 cos 2 eyIg)an 1  Z  2 cos 2 25  2i sin 25 cos 25 2 2 2 1  Z  2 cos (cos  i . sin ) b¤ 5 5 5 (1  Z ) 3  8 cos 3

tamrUbmnþdWmr½eyIg)an ³

3

2 2 2   (1  Z ) 3   2 cos (cos  i . sin ) 5 5 5   2 6 6 (cos  i . sin )  8 cos 3 5 5 5 dUcenH (1  Z)3  8 cos 3 25 (cos 65  i. sin 65 )

eroberogeday lwm plÁún

. - TMBr½88 -

sikSaKNitviTüaedayxøÜnÉg

cMnYnkMupøic

lMhat;TI19 eK[cMnYnkMupøic Z  4 2(1  i ) k-cUrsresr Z CaTRmg;RtIekaNmaRt . x-KNnab¤sTI# én Z . dMeNaHRsay k-sresr Z CaTRmg;RtIekaNmaRt ³ eyIg)an Z  4 2(1  i ) 2 2    i . )  8(  cos  i . sin ) 2 2 4 4      8 cos(   )  i . sin(   ) 4 4   3 3  8(cos  i . sin ) 4 4 3 3 Z  8 (cos  i . sin ) . 4 4  8 (

dUcenH

x-KNnab¤sTI# én Z eyIgtag Wk Cab¤sTI# én Z  8 (cos 34  i. sin 34 )

tamrUbmnþb¤sTI n : Wk  n r cos(  n2k )  i. sin(  n2k ) 

eroberogeday lwm plÁún



- TMBr½89 -

sikSaKNitviTüaedayxøÜnÉg

cMnYnkMupøic

3 3   2 k   2 k     4 4 3 eyIg)an Wk  8 cos ( 3 )  i. sin( 3 )     3  8k 3  8k )  i . sin(  2 [cos( )] 12 12 -ebI k  0 : W0  2 (cos 4  i. sin 4 ) 11 11 -ebI k  1 : W1  2(cos 12  i sin 12 ) -ebI k  2 : W2  2 (cos 1912  i. sin 1912 ) .

eroberogeday lwm plÁún

- TMBr½90 -

sikSaKNitviTüaedayxøÜnÉg

cMnYnkMupøic

lMhat;TI20 eKeGay f z   z 3  2 3  i .z 2  41  i 3 .z  8i k>cUrbgðajfa z ¢ f z   z  2i z 2  2 3z  4 x>edaHRsaysmIkar f z   0 kñúgsMNMukMupøic . dMeNaHRsay k> bgðajfa z ¢ : f z   z  2i z 2  2 3z  4 eyIgman f z   z  2i .z 2  2 3z  4 edayBnøatGnuKmn_enHeyIg)an ³ f ( z )  z 3  2 3z  4z  2i( z 2  2 3z  4)  z 3  2 3 .z 2  4z  2iz 2  4 3iz  8i  z 3  2 3  i .z 2  41  i 3 .z  8i

dUcenH z ¢ f z   z  2i z 2  2

3z  4

Bit

 .

x> edaHRsaysmIkar ebI f z   0 naMeGay z  2i .z 2  2 3 .z  4  0 eKTajb¤s z  2i ehIy , z 2  2 3 .z  4  0 , '  3  4  1  i 2 naMeGay z 1  3  i , z 2  3  i .

eroberogeday lwm plÁún

- TMBr½91 -

sikSaKNitviTüaedayxøÜnÉg

cMnYnkMupøic

lMhat;TI21 eK[cMnYnkMupøic z  cos 47  i. sin 47 . cUrsresr (1  z )4 CaragRtIekaNmaRt . dMeNaHRsay sresr (1  z )4 CaragRtIekaNmaRt³ eKman z  cos 47  i. sin 47 eK)an

4 4  i . sin 7 7 2 2 2 1  z  2 cos 2  2i . sin cos 7 7 7 2 2 2  2 cos (cos  i . sin ) 7 7 7 1  z  1  cos

tamrUbmnþdWmr½eKGacsresr ³

4

2 2 2   (1  z )   2 cos (cos  i . sin ) 7 7 7   2  8 8   16 cos4  i . sin   cos 7  7 7  4

dUcenH

(1  z )4  16 cos 4

eroberogeday lwm plÁún

2  8 8   i . sin   cos 7  7 7 

.

- TMBr½92 -

sikSaKNitviTüaedayxøÜnÉg

lMhat;TI22 eK[cMnYnkMupøic z 1  1  i nig k-cUrsresr z 1

,z2

cMnYnkMupøic

z2 

6  i. 2 2

nig zz1 CaTMrg;RtIekaNmaRt . 2

x-cUrsresr zz1 CaTMrg;BiCKNit . 2

K-eRbIlTæplxagelIcUrTajrktMélR)akdén cos 12 nig sin 12 . dMeNaHRsay k-sresr z 1 , z 2 nig zz1 CaTMrg;RtIekaNmaRt 2

eKman z 1  1  i eday r  12  12  2 eK)an z1  2 22  i. 22   2 cos 4  i. sin 4  .

    eKman z 2  6 2i. 2  2 . 32 i. 2  3   1   z 2  2  i .   2  cos  i . sin  . 2 6 6   2    2  cos  i . sin    4  cos  i . sin eK)an zz1   4 .  12 12  2 2  cos  i . sin  6 6 

eroberogeday lwm plÁún

- TMBr½93 -

sikSaKNitviTüaedayxøÜnÉg

cMnYnkMupøic

x-sresr zz1 CaTMrg;BiCKNit 2

1 i 21  i  6  i . 2  eK)an  6  i . 2  6  i . 2  6  i . 2  2 2 6  i . 2  i . 6  2   62  6  2   i. 6  2   4 dUcenH zz1  6 4 2  i. 6 4 2 . 2 K-TajrktMélR)akdén cos 12 nig sin 12 z1  z2

tamsMrayxagelIeKman ³

z1 z1   6 2 6 2  cos  i . sin   i. ni g z2 12 12 z2 4 4 eKTaj cos 12  i. sin 12  6 4 2  i. 6 4 2 dUcenH cos 12  6 4 2 nig sin 12  6 4 2 .

eroberogeday lwm plÁún

- TMBr½94 -

sikSaKNitviTüaedayxøÜnÉg

cMnYnkMupøic

lMhat;TI23 eK[sIVúténcMnYnkMupøic (Z n ) kMnt;eday

 1 i 3  Z 0  2  1 Z   n  1 2  Z n  | Z n |  ; n  IN

¬ | Z n | Cam:UDulén Z n ¦. snμtfa Z n   n (cos n  i. sin n ) , n  IN Edl  n  0 , n ; n  IR . k-cUrrkTMnak;TMngrvag n nig  n1 ehIy n nig  n1 . x-rkRbePTénsIVút (n ) rYcKNna n CaGnuKmn_én n . K-cUrbgðajfa  n   0 cos  0 cos 21 cos 22 .... cos  n2 1 rYcbBa¢ak; n GnuKmn_én n . dMeNaHRsay k-rkTMnak;TMngrvag n nig  n1 ehIy n nig  n1 eyIgman Z n   n (cos n  i. sin n ) naM[ Z n1   n1 (cos  n1  i sin  n1 ) eday Z n  1  12 ( Z n  | Z n | ) ehIy | Z n |   n eK)an

 n 1 (cos  n 1  i sin  n 1 ) 

eroberogeday lwm plÁún

1  n (cos  n  i. sin  n )   n  2

- TMBr½95 -

sikSaKNitviTüaedayxøÜnÉg

cMnYnkMupøic

1  n 1 (cos  n 1  i . sin  n 1 )   n (1  cos  n  i . sin  n ) 2     n 1 (cos  n 1  i . sin  n 1 )   n cos n (cos n  i . sin n ) 2 2 2 eKTaján n  1   n cos 2n nig  n  1  2n dUcenH  n  1  2n nig n  1   n cos 2n .

x-RbePTénsIVút (n ) nig KNna n CaGnuKmn_én n tamsRmayxagelIeyIgman  n  1  12  n naM[ n CasIVútFrNImaRtmanersug esμI q  12 . tamrUbmnþ  n   0  q n 1 i Z (cos i sin ) eday 0   0  0   0  2 eKTaj)an  0  1 ;  0  3 dUcenH

n 

 1 . n 3 2

eroberogeday lwm plÁún

3

   cos  i . sin 3 3

.

- TMBr½96 -

sikSaKNitviTüaedayxøÜnÉg

cMnYnkMupøic

K-bgðajfa n   0 cos  0 cos 21 cos 22 .... cos 2n tamsRmayxagelIeKman n  1   n cos 2n

b¤ eK)an

n  1 n  cos n 2 k  n 1   k   k  n 1   k 1    cos( )      2  k 0   k  k 0  n 1 2 n  1  cos  0 . cos . cos ......... cos 0 2 2 2

1   cos 2 .... cos n  1 . 2 2 2 mü:ageToteyIgman sin  n  2 sin 2n cos 2n  2 sin n  1 cos 2n eKTaj cos 2n  12 . sinsinn n1 sin  n  1 1 sin  0 sin 1 1 sin  0  . . .....   . ehtuenH n n sin  sin  n sin  n 2 2 sin  n 1 2  sin dUcenH  n  1n  31  n 31 . 1 1 . 2 sin . ) 2 sin( . ) n 3 2 3 2n

dUcenH

 n   0 cos  0 cos

eroberogeday lwm plÁún

- TMBr½97 -

sikSaKNitviTüaedayxøÜnÉg

cMnYnkMupøic

lMhat;TI24 eK[cMnYnkMupøic z  x  i.y Edl x nig y CaBIrcMnYnBit . cUrkMnt;témø x nig y ebIeKdwgfa ³ 10 ( 3  2i ) z  (1  3i ) z  ¬ z CacMnYnkMupøicqøas;én z ¦ . 2i dMeNaHRsay eKman (3  2i ) z  (1  3i ) z  210 i eday z  x  i.y naM[ z  x  i.y eK)an (3  2i )(x  iy )  (1  3i )(x  iy )  210 i 3x  3iy  2ix  2y  x  iy  3ix  3y 

10( 2  i ) 5

(4x  y )  i .( 5x  2y )  4  2i

4x  y  4  5x  2y  2 4 1 4 1 D  8  5  3 , Dx   82 6 5 2 2 2 4 4 Dy   8  20  12 5 2 Dy Dx 6 12 x  2 ,y    4 D 3 D 3

eKTaj)an eKman nig eK)an dUcenH

x  2 , y  4

eroberogeday lwm plÁún

. - TMBr½98 -

sikSaKNitviTüaedayxøÜnÉg

cMnYnkMupøic

lMhat;TI25

eK[ z ; z CacMnYnkMupøicEdl | z || z | r  0 . bgðajfa  rz zzz    rz zzz   r1 1

2

1

2

2

1



2

2

2

1 2

1





2

2

1 2



2

dMeNaHRsay

bgðajfa ³ 2

2

 z1  z 2   z1  z 2  1  2    2   2 r  r  z1z 2   r  z 1z 2 

tag z  r(cos 2x  i sin 2x) nig z Edl x  IR ; y  IR . eK)an ³ 1

2

 r(co2y  i sin 2y )

z1  z 2 r [ (cos 2x  cos 2y )  i(sin 2x  sin 2y ) ]  r 2  z1z 2 r 2  r 2 [ cos( 2x  2y )  i . sin( 2x  2y ) ] 2 cos( x  y ) cos( x  y )  2i sin( x  y ) cos( x  y ) r [ 2 cos 2 ( x  y )  2i sin( x  y ) cos( x  y ) ] 1 cos( x  y )  . r cos( x  y ) z1  z 2 1 sin( y  x )  r 2  z1z 2 r sin( y  x ) 

dUcKñaEdr eK)an ³

eroberogeday lwm plÁún

- TMBr½99 -

sikSaKNitviTüaedayxøÜnÉg 2

cMnYnkMupøic 2

 z1  z 2   z1  z 2  1  cos 2 ( x  y ) sin 2 ( y  x )   2    2   2  2   2 r  z z r z z r cos ( x y ) sin ( y x )          1 2 1 2 cos 2 ( x  y ) 2 2 cos (x  y )  1  cos ( x  y ) 2 cos ( x  y ) sin 2 ( y  x ) 2 2  sin ( x  y ) sin (x  y )  1 2 sin ( y  x ) cos 2 ( x  y ) sin 2 ( y  x) 2 2  cos ( x  y )  sin (x  y )  1  2 2 cos ( x  y ) sin ( y  x )

eday

eRBaH

ehIy

eRBaH

dUcenH

2

2

 z1  z 2   z1  z 2  1   2  2    2 r  r  z1z 2   r  z 1z 2 

eroberogeday lwm plÁún

.

- TMBr½100 -

sikSaKNitviTüaedayxøÜnÉg

cMnYnkMupøic

lMhat;TI26

eKyk z

; z 2 ; ....; z n

1

CacMnYnkMupøicEdlepÞógpÞat;TMnak;TMng

(k  1)z k  1  i(n  k )z k  0 ; k  0 , 1 , 2 , ... , n  1

k-kMnt; z ebIeKdwgfa z  z  z  ....  z  2 x-cMeBaHtémø z Edl)ankMnt;xagelIcUrbgðajfa ³ 0

0

1

2

n

n

0

( 3n  1)n | z 0 |  | z1 |  | z 2 | .... | z n |  n! 2

2

2

2

dMeNaHRsay

k-kMnt; z ebIeKdwgfa z  z  z  ....  z eKman (k  1)z  i(n  k )z  0 eK)an zz  i. nk  k1 0

0

k 1

1

2

n

 2n

k

k 1

k ( p  1)



k 0

 z k  1  p  1 n  k       i .  z k 1    k  k  0 zp n!  i pCpn ; Cpn  z0 p!(n  p )!

eKTaj z

p

eday z

 z1  z 2  ....  z n  2

0

 i p z 0 Cpn ; p  0 , 1 , 2 , ....

eroberogeday lwm plÁún

n

b¤  (z )  2 n

p0

n

p

- TMBr½101 -

sikSaKNitviTüaedayxøÜnÉg

cMnYnkMupøic

man  (z )  z  C i  z (1  i) eK)an z (1  i)  2 eKTaj z  (1 2 i)  (1  i) dUcenH z  (1  i) . x-cMeBaHtémø z Edl)ankMnt;xagelIcUrbgðajfa ³ n

n

p

p0

0

p0

n

n

p p n

0

n

0

n

n

0

n

n

0

0

( 3n  1)n | z 0 |  | z1 |  | z 2 | .... | z n |  n! 2

2

2

2

edayGnuvtþn_vismPaB AM  GM eyIg)an z 0 2  z1 2  ...  z n 2  z 0

2

(C )

 z 0 2 Cn2n  2n .

0 2 n

 (C1n )2  ....  (Cnn )2



( 2n )! n! n!

2n  2n( 2n  1)( 2n  2)....(n  1)) n! 2n  2n  ( 2n  1)  ( 2n  2)  ....  (n  1)     n!  n 

n

( 3n  1)n  n!

dUcenH

( 3n  1)n | z 0 |  | z1 |  | z 2 | .... | z n |  n! 2

2

eroberogeday lwm plÁún

2

2

.

- TMBr½102 -

sikSaKNitviTüaedayxøÜnÉg

cMnYnkMupøic

lMhat;TI27

eK[ z

z1  z 2  z 3

CacMnYnkMupøicedaydwgfa ³ z z z z z z 0 .

cUrbgðajfa

| z1 || z 2 || z 3 |

1

; z2 ; z3

1 2

2 3

3 1

dMeNaHRsay

bgðajfa | z || z || z | eKman z  z  z  z z  z z  z z  0 (1) z  z   z eKTaj z z  z (z  z )  0 (2)  yk (1) CMnYskñúg (2) eyIg)an ³ z z  z  0 naM[ | z | | z | .| z | dUcKñaEdr | z | | z | . | z | nig | z | | z | . | z eyIg)an ³ 1

1

2

2

1

3

3

1 2

2

1 2

3 1

3

3

1

2

2

1 2

2 3

2

3

3

1

2

2

2

2

1

3

1

2

3

|

| z 1 |2  | z 2 |2  | z 3 |2 | z1 || z 2 |  | z 2 || z 3 |  | z 3 || z1 |

b¤ (| z |  | z |) dUcenH | z || z

2

1

2

1

2

 (| z 2 |  | z 3 |)2  (| z 3 |  | z1 |)2  0 || z 3 |

eroberogeday lwm plÁún

.

- TMBr½103 -

sikSaKNitviTüaedayxøÜnÉg

cMnYnkMupøic

lMhat;TI28

cUrbgðajfa

6z  i  1 2  3iz

luHRtaEt

|z|

1 3

dMeNaHRsay

bgðajfa | z |  13 eKman 26z 3izi  1 lkçxNÐ 2  3iz  0 b¤ z  23i eK)an | 6z  i || 2  3iz |

dUcenH

| 6z  i |2 | 2  3iz |2 (6z  i )(6 z  i )  ( 2  3iz )( 2  3iz ) 36zz  6iz  6iz  1  4  6iz  6iz  9zz 27 zz  3 1 zz  9 1 | z |2  9 1 |z|  3

.

eroberogeday lwm plÁún

- TMBr½104 -

sikSaKNitviTüaedayxøÜnÉg

cMnYnkMupøic

lMhat;TI29

eK[ z ; z ; z ; ....; z CacMnYnkMupøicEdlmanm:UDúlesμI 1 . eKtag Z    (z )     ( z1 )  .   cUrbgðajfa 0  Z  n 1

2

3

n

n

n

k

k 1

k 1

k

2

dMeNaHRsay

bgðajfa 0  Z  n eday z ; z ; z ; ....; z CacMnYnkMupøicEdlmanm:UDúlesμI 1 enaHeKGactag z  cos x  i. sin x Edl x  IR ; k  1 , 2 , 3 ,...., n eK)an Z    (z )     ( z1 )  2

1

2

3

n

k

k

k

k

n



n

k 1 n

k

 k 1

k



n

 cos xk  i. sin xk    cos xk  i sin xk 

k 1

k 1

2

2

 n   n     cos xk     sin xk   0  k 1   k 1 

eK)an Z  0 müa:geTottamvismPaB Cauchy  Schwartz 2



n  n    cos xk   n  cos 2 xk  k 1  k 1

eroberogeday lwm plÁún

 - TMBr½105 -

sikSaKNitviTüaedayxøÜnÉg

cMnYnkMupøic

  nig eKTaj Z  n  cos x   n  (sin 2

n  n    sin xk   n  sin 2 xk  k 1  k 1 n

Zn

n

2

k

k 1 n

k 1

2

xk )

 cos2 xk  sin 2 xk 

k 1

dUcenH

Z  n.n  n 2

. sMKal; ³ eKGacRsay Z  n tammYyrebobeTotdUcxageRkam eday | z |  1 enaH z  z1 RKb; k  1 ; 2 ;.....; n 0  Z  n2

2

k

k

k

eK)an

 n   n 1  Z    ( z k )     ( )   k 1   k 1 z k   n   n   n   n     z k     zk     z k     z k   k 1   k 1   k 1   k 1 



n

 z k 

k 1

2

2

 n     | zk |  n2  k 1 

eKTaj)an Z  n . 2

eroberogeday lwm plÁún

- TMBr½106 -

sikSaKNitviTüaedayxøÜnÉg

cMnYnkMupøic

lMhat;TI30

eK[cMnYnkMupøic z Edl | z | 1 . cUrbgðajfa ³ 2  |1 z |  |1 z |  4 . 2

dMeNaHRsay

bgðajfa 2  | 1  z |  | 1  z |  4 tag z  cos t  i. sin t eK)an | 1  z | (1  cos t )  sin t  2 | sin 2t | ehIy | 1  z | (1  cos 2t )  sin 2t  2 | cos t | 2

2

2

2

2

2

t  2 | 1  2 sin 2 | 2 t t   | 1  z |  | 1  z 2 | 2  | sin |  | 1  2 sin 2 |  2 2   t x  sin ;  1  x  1 f 2

eK)an ehIytagGnuKmn_ edayyk kMnt;eday f (x) | x |  | 1  2x | Edl  1  x  1 cMeBaH  1  x  1 eK)an 22  f (x)  2 dUcenH 2  | 1  z |  | 1  z |  4 . 2

2

eroberogeday lwm plÁún

- TMBr½107 -

sikSaKNitviTüaedayxøÜnÉg

cMnYnkMupøic

lMhat;TI31 eK[cMnYnkMupøic Z  (cos x  1 )  i.(sin cos x Edl x CacMnYnBit. cUrkMnt´rkm¨UDulGb,brmaéncMnYnkMupøicen¼ ? dMeNaHRsay rkm¨UDulGb,brmaéncMnYnkMupøic 2

2

x

1 ) sin 2 x

1 2 1 2 2 )  (sin x  ) 2 2 cos x sin x 1 2 1 2 2 f ( x )  (cos 2 x    ) (sin x ) 2 2 cos x sin x 1 1 4 sin x 2  cos 4 x  2     cos 4 x sin 4 x 1 1 )  4  (cos 4 x  sin 4 x )  ( 4  4 sin x cos x cos 4 x  sin 4 x 4 4  4  (cos x  sin x )  ( ) 4 4 sin x cos x

eyIgán tag

2

| Z | (cos 2 x 

eroberogeday lwm plÁún

- TMBr½108 -

sikSaKNitviTüaedayxøÜnÉg

cMnYnkMupøic

f ( x )  4  (cos 4 x  sin 4 x )(1 

1 ) 4 4 sin x cos x





 4  (cos 2 x  sin 2 x ) 2  2 sin 2 x cos 2 x (1 

16 ) sin 4 2x

1 16  4  (1  sin 2 2x )(1  ) 4 2 sin 2x

edayeKman

sin 2 2x  1

naM[

1 1 1  sin 2 2x  2 2

16  17 4 sin 2x eKTaj 4  (1  1 sin 2 2x)(1  164 )  4  17  25 2 2 2 sin 2x eyIgán f ( x)  4  (1  1 sin 2 2x)(1  164 )  25 2 2 sin 2x 5 5 2 eday | Z | f (x) eKTaján | Z |   2 2 dUcen¼m¨UDulGb,brmaén Z KW | Z |min  5 2 . 2

nig

1

eroberogeday lwm plÁún

- TMBr½109 -

sikSaKNitviTüaedayxøÜnÉg

cMnYnkMupøic

lMhat;TI32

eK[ cUrbgðajfa

n

   1  1 A  i   i      3  3 A  i.

2 n 1 n

( 3)

. sin

n 3

n

, n  IN

.

cMeBaHRKb; n  IN .

dMeNaHRsay

bgðajfa

A  i.

2 n 1 ( 3)

n

. sin

n 3

cMeBaHRKb; n  IN .

eyIgman , n  IN tag Z  13  i  1  i3. 3  23  12  i 23   23  cos 3  i.sin 3      tamrUbmnþdWmr½eK)an Z   1  i   2  cos n3  i. sin n3  ( 3)    3  ehIy Z  2  cos n3  i. sin n3  ( 3)   eKTaj A  2  cos n3  i. sin n3   2  cos n3  i. sin n3  n

 1   1   i   i  A   3   3 

n

n

n

n

n

n

n

n

n

n

( 3 )n 



( 3 )n 



n 1

n n n n  2 n       i . sin i . . sin cos i . sin cos   3 3 3 3  3 ( 3 )n ( 3 )n  2n

dUcenH

A  i.

2 n 1 ( 3)

n

eroberogeday lwm plÁún

. sin

n 3

.

- TMBr½110 -

sikSaKNitviTüaedayxøÜnÉg

cMnYnkMupøic

lMhat;TI33  

kñúgbøg;kMupøic (O , i , j ) eK[bYncMnuc A , B , C , D EdlmanGahVikerogKña Z  1  6i , Z  4  5i , Z  5  4i nig Z  2  3i . cUrRsayfactuekaN ABCDcarikkñúgrgVg;mYyEdleKnwgbBa¢ak;p©it nig kaMrbs;va . A

B

D

C

dMeNaHRsay

RsayfactuekaN ABCDcarikkñúgrgVg; eyIgtag (c): x  y  ax  by  c  0 CasmIkarrgVg;carikeRkARtIekaN ABC . eyIg)an A  (c) naM[ 1  6  a  6b  c  0 b¤ a  6b  c  37 (1) B  (c) naM[ 4  5  4a  5b  c  0 b¤ 4a  5b  c  41 (2) C  (c) naM[ ( 2)  ( 3)  2a  3b  c  0 b¤  2a  3b  c  13 (3) 2

2

2

2

2

2

2

eroberogeday lwm plÁún

2

- TMBr½111 -

sikSaKNitviTüaedayxøÜnÉg

eyIg)anRbB½næsmIkar

cMnYnkMupøic

a  6b  c  37  4a  5b  c  41   2a  3b  c  13 

bnÞab;BIedaHRsayRbB½næenHeK)ancemøIuy a  2 , b  2 , c  23 . smIkarrgVg;carikeRkARtIekaN ABC Gacsresr ³ (c) : x 2  y 2  2x  2y  23  0

b¤ (c) : (x  1)  (y  1)  25 müa:geTotedayykkUGredaen D CYskñúgsmIkar 2

2

(c) :( 2  1) 2  ( 3  1) 2  25

vaepÞógpÞat;enaHnaM[ D (c) . edaybYncMnuc A , B , C , D sßitenAelIrgVg;mansmIkar (c) : ( x  1)  ( y  1)  25 EtmYy enaHnaM[ctuekaN ABCDcarikkñúgrgVg; (c) manp©it I( 1 , 1 ) nig kaM R  5 . 2

2

eroberogeday lwm plÁún

- TMBr½112 -

sikSaKNitviTüaedayxøÜnÉg

cMnYnkMupøic

lMhat;TI34

eK[ z nig z CaBIrcMnYnkMupøic . cUrbgðajfa | 1  z z |  | z  z |  1

2

1 2

1

| z 1  1 || z 2  1 | 2

2

2

dMeNaHRsay

bgðajfa | 1  z z |  | z  z |  tamvismPaBRtIekaNeK)an ³ 1 2

1

2

| z 1  1 || z 2  1 | 2

2

| 1  z 1z 2 |  | z 1  z 2 |  | 1  z 1z 2  z 1  z 2 |

nig | 1  z z |  | z  z |  | 1  z z  z  z | eK)an | 1  z z |  | z  z |  | (1  z z )  (z  z ) | eday (1  z z )  (z  z )  1  z  z  z z  (1  z eK)an | 1  z z |  | z  z |  | (1  z )(1  z ) | 1 2

1

2

1 2

1

2

2

1 2

1

2

1 2

2

2

2

2

1

2

1

2

1

2

1

2

2

1 2

2

1 2

2

1

2

1

2

2

2

2 1

)(1  z 2 ) 2

2

2

| 1  z1z 2 |  | z1  z 2 | 2  | 1  z12 || 1  z 2 2 | dUcenH | 1  z1z 2 |  | z1  z 2 | 2  | (1  z1z 2 )2  (z1  z 2 )2 |

eroberogeday lwm plÁún

- TMBr½113 -

sikSaKNitviTüaedayxøÜnÉg

cMnYnkMupøic

lMhat;TI35 eK[ z nig z CacMnYnkMupøicBIr . cUrRsayfa | z  z |  | z  z |  2| z |  | z |  dMeNaHRsay Rsayfa | z  z |  | z  z |  2| z |  | z |  eKman | z  z |  (z  z )( z  z ) 1

2

2

1

2

2

1

2

1

2

2

2

2

1

2

1

2

2

2

2

1

2

2

1

2

1

2

1

2

| z 1  z 2 |2  z 1 z1  z 1 z 2  z1z 2  z 2 z 2

| z 1  z 2 |2 | z 1 |2  z 1 z 2  z1z 2  | z 2 |2 1

ehIy

| z 1  z 2 |2  ( z 1  z 2 )( z1  z 2 )

| z 1  z 2 |2  z 1 z1  z 1 z 2  z1z 2  z 2 z 2 | z 1  z 2 |2 | z 1 |2  z 1 z 2  z1z 2  | z 2 |2

bUkTMnak´TMng

1

nig

2 

eKán ½

| z 1  z 2 |2  | z 1  z 2 |2  2  | z 1 |2  | z 2 |2 

eroberogeday lwm plÁún

2 

.

- TMBr½114 -

sikSaKNitviTüaedayxøÜnÉg

cMnYnkMupøic

lMhat;TI36 eK[ z nig z CacMnYnkMupøicBIrEdl | z nig z .z  1 . cUrRsayfa 1z z zz CacMnYnBitmYy . dMeNaHRsay Rsayfa 1z z zz CacMnYnBitmYy 1

1

2

1

|  | z2 |  1

2

1

2

1 2

1

2

1 2

tag

eday eKán eday

z1  z 2 1  z1z 2

ena¼

Z

z 1 .z1  | z 1 |2  1

ena¼

Z

z1  z 2 1  z1 .z 2 1 z1  z1

ehIydUcKña

z2 

1 z2

1 1  z  z1 z z2 Z 1  2 Z 1 1 z 2 z1  1 1 . z1 z 2 z  z2 ZZ Z 1 1  z 1z 2

ena¼

eroberogeday lwm plÁún

CacMnYnBit .

- TMBr½115 -

sikSaKNitviTüaedayxøÜnÉg

cMnYnkMupøic

lMhat;TI37 eK[BhuFa P(x)  (x sin a  cos a) Edl n  IN * cUrrksMNl´énviFIEckrvag P(x) nwg x  1 . dMeNaHRsay sMNl´énviFIEck tag R(x) CasMNl´énviFIEckrvag P(x) nwg x  1 -ebI n  1 ena¼ P(x)  x sin a  cos a dUcen¼ R(x)  x sin a  cos a CasMnl´énviFIEck . -ebI n  2 eKán P(x)  (x  1)Q(x)  R(x) Edl Q(x) CaplEck nwg R(x)  Ax  B eKán (x sin a  cos a)  (x  1)Q(x)  Ax  B ebI x  i ena¼ (i sin a  cos a)  Ai  B ¦ cos(na)  i. sin(na)  B  i.A eKTaj A  sin(na) nig B  cos(na) dUcen¼ R(x)  x sin(na)  cos(na) . n

2

2

2

n

2

n

eroberogeday lwm plÁún

- TMBr½116 -

sikSaKNitviTüaedayxøÜnÉg

cMnYnkMupøic

lMhat;TI38

eK[cMnYnkMupøic z  2  cos   i 2  sin  Edl   IR   kñúgbøg;kMpøic (o , i , j ) eKehA M CacMnucrUbPaBén z . cUrkMNt;témøtUcbMput nig FMbMputén r  OM ? dMeNaHRsay

kMNt;témøtUcbMput nig FMbMputén r  OM eKman ³ OM 2  a 2  b 2  ( 2  cos  ) 2  ( 2  sin  ) 2 OM 2  4  cos   sin 

b¤ OM 

4  cos   sin 

eK)an r  4  cos   sin  tamvismPaB Cauchy  Schwarz eKman ³ | cos   sin  | 12  12 cos 2   sin 2   2

b¤  2  cos   sin   2 eKTaj 4  2  r  4  2 RKb;   IR dUcenH rmin  4  2 nig rmax  4  2 .

eroberogeday lwm plÁún

- TMBr½117 -

sikSaKNitviTüaedayxøÜnÉg

cMnYnkMupøic

lMhat;TI39

eK[cMnYnkMupøic z1 , z 2 , z 3 ehIyepÞógpÞat;TMnak;TMng ³ | z 1 || z 2 || z 3 | 1 nig

z 32 z12 z 22   1 0 z 2 z 3 z 1z 3 z 1z 2

cUrRsayfa | z1  z 2  z 3 |  { 1 , 2 } . dMeNaHRsay Rsayfa | z1  z 2  z 3 |  { 1 , 2 } eKman

z 32 z12 z 22   1 0 z 2 z 3 z 1z 3 z 1z 2

eK)an z13  z 2 3  z 3 3  z1z 2 z 3  0 b¤ z13  z 2 3  z 3 3  3z1z 2 z 3  4z1z 2 z 3 tag z  z1  z 2  z 3 eK)an ³ z 3  3z ( z 1 z 2  z 2 z 3  z 1 z 3 )  4z 1 z 2 z 3   1 1 1 z  z 1 z 2 z 3  3z (   )  4  z1 z 2 z 3  3

z 3  z 1z 2 z 3 3z( z 1  z 2  z 3 )  4



z 3  z 1 z 2 z 3 ( 3 z .z  4 )  z 1 z 2 z 3 3 | z | 2  4



eK)an | z |3 | z1z 2 z 3 (3 | z |2 4) | eroberogeday lwm plÁún

- TMBr½118 -

sikSaKNitviTüaedayxøÜnÉg

b¤ | z |3  | 3 | z |2 4 | -ebI 3 | z |2 4  0 b¤ | z |  eK)an | z |3  3 | z |2 4

cMnYnkMupøic 2 3

| z |3 3 | z |2 4  0 (| z | 1)(| z | 2) 2  0 | z | 2 2 2 -ebI 3 | z | 4  0 b¤ | z |  3 eK)an | z |3  (3 | z |2 4) | z |3 3 | z |2 4  0 (| z | 1)(| z | 2) 2  0 | z | 1

dUcenH | z1  z 2  z 3 |  { 1 , 2 } .

eroberogeday lwm plÁún

- TMBr½119 -

sikSaKNitviTüaedayxøÜnÉg

cMnYnkMupøic

lMhat;TI40

eK[cMnYnkMupøic z1 nig z 2 Edl | z1 || z 2 | 1 cUrRsayfa | z1  1 |  | z 2  1 |  | z1z 2  1 | 2 dMeNaHRsay

Rsayfa | z1  1 |  | z 2  1 |  | z1z 2  1 | 2 tamvismPaBRtIekaN | a |  | b |  | a  b | eK)an ³ | z 2  1 |  | z 1z 2  1 || z 2  1  z 1 z 2  1 |

| z 2  1 |  | z 1z 2  1 || z 2 || 1  z 1 || 1  z 1 |

ehIy | z1  1 |  | 1  z1 || (z1  1  1  z1 ) | 2 dUcenH | z1  1 |  | z 2  1 |  | z1z 2  1 | 2 .

eroberogeday lwm plÁún

- TMBr½120 -

sikSaKNitviTüaedayxøÜnÉg

cMnYnkMupøic

lMhat;TI41

eK[sIVútcMnYnBit (an ) kMNt;eday ³

a 1  1 , a 2  1  a n  2  a n 1  a n , n  1 , 2 , 3 , ... 1 i 3 eKtagsIVútcMnYnkMupøic z n  an1  an . 2 k> cUrRsayfa z n1  1  i 3 z n cMeBaHRKb; n  1 . 2 x> cUrdak; 1  i 3 CaTRmg;RtIekaNmaRtrYcTajrk z n CaGnuKmn_ 2 én n .

K> TajrktYTUeTAénsIVút an . etI (an ) CasIVútxYbb¤eT ? dMeNaHRsay

k> Rsayfa z n1  1  i

3

zn

cMeBaHRKb; n  1

2 eKman z n  an1  1  i 3 an 2 1 i 3 eK)an z n1  an 2  a n 1 2 eday an 2  an1  an

eroberogeday lwm plÁún

- TMBr½121 -

sikSaKNitviTüaedayxøÜnÉg

cMnYnkMupøic

1 i 3 eK)an z n1  an1  an  a n 1 2 1 i 3 a n 1  a n  2 1 i 3 2 (a n  1  an )  2 1 i 3 1 i 3 1 i 3 (a n  1   an ) 2 2 1 i 3 zn . dUcenH z n1  2 x> dak; 1  i 3 CaTRmg;RtIekaNmaRt ³ 2 eK)an 1  i 3  1  i 3  cos   i sin  2 2 2 3 3 Tajrk z n CaGnuKmn_én n ³

eday z n1  1  i

2

3

zn

enaH (z n ) CasIVútFrNImaRténcMnYn

   cos  i sin 2 3 3 1 i 3 1 i 3   z1  a 2  a1   cos  i sin 2 2 3 3

kMupøicEdlmanersug q  1  i nigtY

eroberogeday lwm plÁún

3

- TMBr½122 -

sikSaKNitviTüaedayxøÜnÉg

cMnYnkMupøic

tamrUbmnþ z n  z1  qn1  (cos   i sin  )n 3

3

tamrUbmnþdWm½reK)an z n  cos n  i sin n . 3 3 K> TajrktYTUeTAénsIVút an eKman z n  an1  1  i 3 an 2

eK)an z n  (an1  an )  i

3 a n (1) 2 2 eday z n  cos n  i. sin n (2) 3 3 tamTMnak;TMng (1) & (2) eK)an 3 an  sin n 2 3 dUcenH an  2 sin n . 3 3 2 ehIy (an ) CasIVútxYbEdlmanxYb p    6 . 3

eroberogeday lwm plÁún

- TMBr½123 -

sikSaKNitviTüaedayxøÜnÉg

cMnYnkMupøic

lMhat;TI42

eK[sIVúténcMnYnkMupøic (z n ) kMNt;eday ³ 2 3i 3i 2 3i z1  zn  nig z n1  2 2 2 Edl n  1 , 2 , 3 , ... . k> tag w n  z n  1 . bgðajfa (w n ) CasIVútFrNImaRténcMnYn kMupøic rYcKNna w n CaGnuKmn_én n edaysresrlTæplCaTRmg; RtIekaNmaRt . x> Tajbgðajfa z n  2 cos n (cos n  i sin n ) . 12

dMeNaHRsay

12

12

k> bgðajfa (w n ) CasIVútFrNImaRténcMnYnkMupøic ³ eKman w n  z n  1 eK)an w n1  z n1  1 3i 2 3i  zn  1 2 2 3i 3i  ( z n  1)  wn 2 2

dUcenH (w n ) CasIVútFrNImaRténcMnYnkMupøic . eroberogeday lwm plÁún

- TMBr½124 -

sikSaKNitviTüaedayxøÜnÉg

cMnYnkMupøic

KNna w n CaGnuKmn_én n ³ eK)an w n  w1  qn1 eday w1  3  i  cos   i sin 

2 6 6 3i   nig q   cos  i sin 2 6 6 eK)an w n  (cos   i sin  )n 6 6 dUcenH w n  cos n  i sin n ¬rUbmnþdWmr½¦ 6 6 n n n x> Tajbgðajfa z n  2 cos (cos  i sin ) 12 12 12 eKman w n  z n  1 enaH z n  1  w n n n z n  1  cos  i sin 6 6 n n n  2 cos 2  2i . sin cos 12 12 12 n n n  2 cos (cos  i sin ) 12 12 12 dUcenH z n  2 cos n (cos n  i sin n ) . 12 12 12

eroberogeday lwm plÁún

- TMBr½125 -

sikSaKNitviTüaedayxøÜnÉg

cMnYnkMupøic

lMhat;TI43

eK[sIVúténcMnYnBit (un ) nig ( v n ) kMNt;eday ³ un  vn  u n 1  2 nig  un  vn v  n 1  2 k> eKBinitüsIVúténcMnYnkMupøic z n  un  i.v n  u1   v   1

2 2 2 2

Edl n  1

. cUrRsayfa (z n ) CasIVútFrNImaRténcMnYnkMupøic rYcKNna z n CaGnuKmn_én n edaysresrlTæplCaTRmg;RtIekaNmaRt . x> sMEdg un nig v n CaGnuKmn_én n . dMeNaHRsay

k> Rsayfa (z n ) CasIVútFrNImaRténcMnYnkMupøic ³ eKman z n  u n  i.v n eK)an z n1  un1  i.v n1

un  vn u  vn  i. n 2 2 2i 2 2i 2  (u n  iv n )  zn 2 1 

dUcenH(z n ) CasIVútFrNImaRténcMnYnkMupøic . eroberogeday lwm plÁún

- TMBr½126 -

sikSaKNitviTüaedayxøÜnÉg

cMnYnkMupøic

KNna z n CaGnuKmn_én n ³ eK)an z n  z1  qn1 Et z1  u1  iv1  2  i

2    cos  i . sin 2 2 4 4   2 2 nig q   i  cos  i. sin 2 4 4 2 eK)an z n  (cos   i sin  )n 4 4 dUcenH z n  cos n  i. sin n ¬rUbmnþdWm½r ¦ 4 4 x> sMEdg un nig v n CaGnuKmn_én n

eKman z n  u n  i.v n eday z n  cos n  i. sin n 4 n dUcenH un  cos 4

eroberogeday lwm plÁún

4

nig

n v n  sin 4

.

- TMBr½127 -

sikSaKNitviTüaedayxøÜnÉg

cMnYnkMupøic

lMhat;TI44

eK[sIVúténcMnYnBit (un ) nig ( v n ) kMNt;eday ³ u 0  1  v0  3

u n  1  u n 2  v n 2 nig   v n 1  2u n v n k> eKBinitüsIVúténcMnYnkMupøic z n  un  i.v n .

Edl n  0 2n

cUrRsayfa z n1  z n rYcTajfa z n  z 0 . x> sMEdg un nig v n CaGnuKmn_én n . 2

dMeNaHRsay 2n

k>Rsayfa z n1  z n rYcTajfa z n  z 0 ³ eKman z n  un  i.v n eK)an z n1  un1  iv n1 2

 u n 2  v n 2  2iu n v n  u n 2  2iu n v n  (iv n ) 2  (u n  iv n ) 2

dUcenH z n1  z n 2 .

eroberogeday lwm plÁún

- TMBr½128 -

sikSaKNitviTüaedayxøÜnÉg

cMnYnkMupøic

müa:geTotebI n  0 enaH z1  z 0 2 ebI n  1 enaH z 2  z12  z 0 4 ebI n  2 enaH z 3  z 2 2  z 08 2k ]bmafavaBitdl;tYTI k KW z k  z 0 2k  1 eyIgnwgRsayfavaBitdl;tYTI k  1 KW z k 1  z 0 2 2k eKman z k 1  z k Ettamkar]bma z k  z 0 2k 2 2k 1 eK)an z k 1  (z 0 )  z 0 Bit . 2n dUcenH z n  z 0 . x> sMEdg un nig v n CaGnuKmn_én n 2n eKman z n  z 0 eday z 0  u 0  iv 0  1  i 3    2(cos  i sin ) 3 3   2n 2n eK)an z n  2 (cos  i sin ) 3 3 n n n 2 2    2 2 (cos  i sin ) 3 3 2n  2n  2n 2n dUcenH un  2 cos ; v n  2 sin . 3 3

eroberogeday lwm plÁún

- TMBr½129 -

sikSaKNitviTüaedayxøÜnÉg

cMnYnkMupøic

CMBUkTI3 lMhat;Gnuvtþn_ !> cUrsresrcMnYnkMupøicxageRkamCaTRmg;BICKNit a  i.b ³ k> (2  5i )(3  i ) x> (1  i )(1  i ) K> (1  i )(1  2i )(1  3i ) X> (2  i )3 g> 4 1 3i c> 44  33ii q>

( 3  2i )(i  1) i 1

13  12i ( 2i  1)2 Q> 6i  8  2  i @> KNna (1  2i )3 nig (3i  4)4

2

C>  11  ii    j> (i 2  31)(ii 3  2 )

#> eK[ x CacMnYnBit . kMNt; x edIm,I[ 13  ii  1x  ii CacMnYnBit . $> eK[cMnYnkMupøic z1  3  2i nig z 2  2  4i . cUrkMNt;rUbPaBén z1  z 2 , z1  z 2 nig 2z 2 ? %> eK[ z1  2  i , z 2  1  2i nig z 2  1  3i k-KNna z13  z 23  z 33 nig z1  z 2  z 3 x-bgðajfa z13  z 23  z 33  3z1z 2z 3

eroberogeday lwm plÁún

- TMBr½130 -

sikSaKNitviTüaedayxøÜnÉg

cMnYnkMupøic

^> eK[cMnYnkMupøicBIr z1  a  i.b nig z 2  b  i.a Edl a nig b CaBIrcMnYnBit . cUrkMnt;témørbs; a nig b edIm,I[ Z1  Z 2  7  5i . &> kMnt;cMnYnBit a nig b edIm,I[ (2  3i ) Cab¤sénsmIkar x 2  ax  b  0 *> kMnt;cMnYnBit p nig q edIm,I[ 1  2i Carwsrbs;smIkar z 3  pz  q  0 . (> k> cUrepÞógpÞat;fa 3  4i  (2  i ) 2 x> edaHRsaysmIkar Z 2  (4  5i ) Z  3  11 i  0 . !0> k> cUrepÞógpÞat;fa  3  4i  (1  2i )2 x> edaHRsaysmIkar Z 2  (2  i ) 2 Z  1  7i  0 . !!> eK[cMnYnkMupøic U  2  3i , V  3  2i nig W  1  5i . cUrepÞógpÞat;fa U 3  V 3  W 3  3U.V.W . !@> kMnt;BIrcMnYnBit x nig y ebIeKdwgfa ³ (1  3i )( x  iy )  ( 2  i )( x  iy )  7  0 . !#> kMnt;BIrcMnYnBit x nig y ebIeKdwgfa (3  2i )x  (1  3i )y  5(1123ii ) . z 2  2z  5

!$> eK[GnuKmn_ f (z )  2 Edl z   i . z 1 cUrKNna f (1  2i ) , f (1  i ) nig f (2  i ) .

eroberogeday lwm plÁún

- TMBr½131 -

sikSaKNitviTüaedayxøÜnÉg

cMnYnkMupøic

!%> eK[ z  1  2i nig U  a  i.b . cUrkMnt;cMnYnBit a nig b edIm,I[ U  1  z  z 2  z 3  z 4 . !^> k>eK[ z  1  i . cUrKNna z 2 . x> KNnaplbUk S  1  z  z 2  z 3  .....  z 2006 . !&> eK[cMnYnkMupøic Z  x  i.y , x  IR , y  IR mankMupøicqøas;tageday Z . cUrkMnt;rktémø x nig y edIm,I[ (1  i ) Z  (3  2i ) Z  2(19  i2i ) !*> eK[cMnYnkMupøic A  13  52ii , B   12 i6i nig C  816ii k> cUrsresr A , B , C CaTRmg;BICKNit . x> cUrepÞógpÞat;fa B 2  4A.C  4(2  i )2 . K> edaHRsaysmIkar A Z 2  B Z  C  0 edaysresrb¤snImYy² CaTRmg;BICKNit . !(> eK[cMnYnkMupøic Z  (x 2  y 2 )  2ixy nig U  3  4i . k> cUrsresr U 3 CaTRmg;BICKNit . x> kMnt;cMnYnBit x nig y edIm,I[ Z  U 3 . @0> k> cUrbgðajfa  77  36i  (2  9i )2 x> edaHRsaysmIkar (2  i ) z 2  (8  i ) z  5(3  i )  0 rYcsresrb¤snImYy²CaTRmg;BICKNit .

eroberogeday lwm plÁún

- TMBr½132 -

sikSaKNitviTüaedayxøÜnÉg

cMnYnkMupøic

@!> cUrKNnab¤skaeréncMnYnkMupøic Z  45  28i . @@> cUrKNnab¤skaeréncMnYnkMupøic Z  40  42i . @#> eK[ Z  11  22  ii . cUrsresr Z CaTRmg;BICKNit rYcKNna Z 2 . @$> eK[cMnYnkMupøic z  a  b.i , a, b  IR . cUrkMnt;témø a nig b edIm,I[ 16 z 2  23 z  2  i . @%> eK[smIkar (E) : z 3  (4  5i )z 2  5(1  3i )  2(7  i )  0 . k> kMnt;cMnYnBit b edIm,I[ z 0  bi Cab¤ssmIkar (E) . x> cUrsresrsmIkar (E) Carag (z  z 0 )(z 2  pz  q)  0 Edl p nig q CacMnYnkMupøicRtUvrk . K> edaHRsaysmIkar (E) kñúgsMNMukMupøic . @^> eK[bIcMnYnkMupøic ³ U    i .x , V    i .y , W    i .z , ,  ,  , x, y , z  IR       0 3 3 3 cUrbgðajfa U  V  W  3U.V.W luHRtaEt x  y  z  0  @&> eK[smIkar (E) : z 3  az 2  bz  c  0 .

cUrrkcMnYnBit a, b, c edIm,I[ z  1 nig eroberogeday lwm plÁún

z  1  2i

.

Cab¤ssmIkar (E) . - TMBr½133 -

sikSaKNitviTüaedayxøÜnÉg

cMnYnkMupøic

@*> eKmansmIkar (E): z 2  3z  4  6i  0 k-kMnt;cMnYnBit b edIm,I[ z 0  b.i Cab¤srbs;smIkar (E) . x-cUredaHRsaysmIkar (E) kñúgsMNMukMupøic . @(> eKmansmIkar (E): z 2  5(1  i )z  3(4  3i )  0 k-cUrepÞógpÞat;fa 48  14i  (7  i ) 2 x-cUredaHRsaysmIkar (E) kñúgsMNMukMupøic . #0> eKmansmIkar (E) : z 2  (a  i.b)z  a  3  5i  0 Edl a , b IR . kMnt;témø a nig b edIm,I[ z1  3  2i Cab¤smYyrbs;smIkar (E) rYccUrkMnt;rkb¤s z 2 mYyeTotcMeBaHtémø a nig b Edl)anrkeXIj . #!> eKmansmIkar (E) : z 3  (5  i )z 2  (10  9i )z  2(1  8i )  0 . k-kMnt;cMnYnBit b edIm,I[ z 0  b.i Cab¤srbs;smIkar (E) . x-cUrsresrsmIkar (E) Carag (z  z 0 )(z 2  pz  q)  0 Edl p nig q CacMnYnkMupøiucBIrEdleKRtUvrk . K-epÞógpÞat;fa  8  6i  (1  3i ) 2 rYcedaHRsaysmIkar (E) kñúgsMNMukMupøic. #@> cUredaHRsaysmIkarxageRkamkñúgsMNMukMupøic ³ a / iz 2  ( 2  3i )z  (1  5i )  0 b / ( 2  i )z 2  5(1  i )z  2( 3  4i )  0 c / (1  i )z 2  (1  7i )z  2( 2  3i )  0

eroberogeday lwm plÁún

- TMBr½134 -

sikSaKNitviTüaedayxøÜnÉg

cMnYnkMupøic

##> begáItsmIkardWeRkTIBIrEdlman  nig  Cab¤skñúgkrNInImYy²xageRkam ³ k> a /   2  11i ,   2  11i b /   2  3i ,   2  3i

x> a /

  3  2i ,   1  3i

b /   2  3i ,   3  2i

K> a /    1  2i

,   3  2i

b/    3  i ,   1 i 3

#$> eKmancMnYnkMupøic   1  3i nig   1  2i . cUrsresrsmIkardWeRkTIBIrmYyEdlman Z1   2   2 nig Z 2   2 . 2 Cab¤s> #%> cUrkMnt;rkcMnYnkMupøicEdlmanm:UDúlesμI 8 ehIykaerrbs;vaCacMnYnnimμitsuTæ. #^> cUrkMnt;BIrcMnYnBit x nig y ebIeKdwgfa ³ 8  9i . (1  i )( x  iy )  ( 3  2i )( x  iy )  1  2i #&> cUrkMnt;BIrcMnYnBit x nig y ebIeKdwgfa ³ 5  13i . ( 3  2i ) ( x  y )  (1  2i )( x  y )  1 i #*> eKmansmIkar (E) : az 2  bz  c  0 , a  0 , a, b, c  IR . cUrbgðajfaebI z 0 Cab¤ssmIkar (E) enaH z 0 k¾Cab¤srbs; (E) Edr . eroberogeday lwm plÁún

- TMBr½135 -

sikSaKNitviTüaedayxøÜnÉg

cMnYnkMupøic

1 i 5  2

1 i 5 #(> eKmancMnYnkMupøic nig   2 . k-cUrbegáItsmIkardWeRkTIBIrmYyEdlman  nig  Cab¤s .

x-eKtag S n   n   n cMeBaHRKb; n  IN . cUrRsaybBa¢ak;TMnak;TMng S n 2  S n1  32 S n  0 ? 10

10

K-edaymin)ac;BnøatcUrKNna N   1  2i 5    1  2i 5      $0> eKmansmIkar (E) : z 2  (  )iz    0 , ,   IR *

.

cUrbgðajfasmIkar (E) b¤sBIrsuTæEtCacMnYnnimμitsuTæEdleKnwgbBa¢ak; . $!> eKmansmIkardWeRkTIBIr (E) : Az 2  Bz  C  0 Edl A  0 ehIy A , B , C CacMnYnkMupøic . k> bgðajfaebI A  C  i.B enaHsmIkar (E) manb¤sBIrkMnt;eday ³ C z 1  i , z 2   i. . A x> bgðajfaebI A  C   i.B enaHsmIkar (E) manb¤sBIrkMnt;eday ³ C z 1  i , z 2  i. . A K> Gnuvtþn_ ³ cUredaHRsaysmIkarxageRkamkñúgsMNMukMupøic ³ a / (1  i )z 2  ( 2  3i )z  2  3i  0 b / (1  2i )z 2  (1  2i )z  (1  i )  0

eroberogeday lwm plÁún

- TMBr½136 -

sikSaKNitviTüaedayxøÜnÉg

cMnYnkMupøic

$@> eK[cMnYnkMupøic Z1  1  2i nig Z 2  3  i . cUrkMnt;rkEpñkBit nigEpñknimμiténcMnYnkMupøic W ebIeKdwgfa W1  Z1

1

1 .  Z2

$#> eKmancMnYnkMupøic  nig  Edl     3  2i nig .  5(1  i ) . cUrkMnt;EpñkBit nig Epñknimμitén Z  12  12 . 



$$> eKmansmIkar (E) : i.Z 2  (2  3i ) Z  5 (1  i )  0 k> kMnt;cMnYnBit a edIm,I[ Z1  a  i Cab¤smYyrbs;smIkar (E) rYcKNna b¤s Z 2 mYyeTotrbs;smIkar . x> cUrkMnt;cMnYnBit p nig q edIm,I[ Zp  Zq  pZ q Z 2 . 1

2

1

2

$%> eKmancMnYnkMupøicBIr Z1 nig Z 2 Edl Z1  Z 2  3  i nig Z1 .Z 2  4  3i . k> cUrbgðajfa (Z1 ) 2  (Z 2 ) 2  0 . x> cUrkMnt;rkEpñkBit nig Epñknimμit én Z  Z14  Z 2 4 . K> cUrepÞógpÞat;fa Z1  Z 2 2  4Z1 .Z 2  (Z1  Z 2 ) 2 rYckMnt;rk Z1 nig Z 2 . $^> eKmancMnYnkMupøic Z1  x 2  i xy nig Z 2  y 2  ixy Edl x, y  IR . cUrkMnt; x nig y edIm,I[ Z1  Z 2  ( 2006  i 2007 ) 2 . eroberogeday lwm plÁún

- TMBr½137 -

sikSaKNitviTüaedayxøÜnÉg

cMnYnkMupøic

$&> eK[cMnYnkMupøic ³ Z 1  (a  b )  i (b  c ) , Z 2  (b  c )  i (c  a )

nig Z 2  (c  a)  i(a  b) Edl a , b , c CabIcMnYnBitxusKña. cUrbgðajfa Z1 3  Z 2 3  Z 3 3  3Z1 .Z 2 .Z 3 $*> cUrKNnab¤skaeréncMnYnkMupøicxageRkam ³ k> a / Z  48  14i b / Z  24  10i x> a / Z  40  42i b / Z  77  36i K> a / Z  55  48i b/ Z  1 i 6 $(> cUrsresrcMnYnkMupøicxageRkamCaTRmg;RtIekaNmaRt ³ k> a / Z  1  i 3 b / Z  2 3  2i x> a / Z  1  i b / Z   3  3i 2 K> a / Z  6 2i 2 X> a / Z  1  i3 g> a / Z  1  2  i

b / Z  2  2i b/Z   2  i 2 b/Z  2 3  i

%0> cUrsresrCaTRmg;RtIekaNmaRténcMnYnkMupøicxageRkam ³ k> a / Z  1  cos 49  i. sin 49 b / Z  1  sin 10  i. cos 10 eroberogeday lwm plÁún

- TMBr½138 -

sikSaKNitviTüaedayxøÜnÉg

x> a / Z  sin 27  i(1  cos 27 ) K> a / Z 

2  2 cos

b / Z  1  i tan

 7

cMnYnkMupøic b / Z  1  cos

   i . 2  2 cos 5 5

4 4  i sin 5 5

X> a / Z   sin 10  i. cos 10

  b / Z  ( 3  2)(cos  i sin ) 8 8   Z  2  2  2  2 cos  i . 2  2  2  2 cos 4 4

g> . %!> cUredaHRsaysmIkarxageRkamrYcsresrb¤snImYy²CaragRtIekaNmaRt ³ b / 2z 2  2z  1  0 k> a / z 2  2z  4  0 x> a / z 2  3 .z  1  0 b / z 2  z  1  0 %@> cUrsresr Z  2  3  i. 2  3 CaragRtIekaNmaRt . %#> cUrsresr Z  2  2  i 2  2 CaragRtIekaNmaRt . %$> k-KNnatémøR)akdén sin 10 , cos 10 nig tan 10 . x-TajrkTRmg;RtIekaNmaRtén Z  1  i. 5  2 2 .

eroberogeday lwm plÁún

- TMBr½139 -

sikSaKNitviTüaedayxøÜnÉg

cMnYnkMupøic

2i 6 %%> eK[cMnYnkMupøic z1  2 nig z 2  1  i k> cUrsresr Z  zz1 CaTRmg; a  b.i . 2 x> cUrsresr z 1 , z 2 nig Z  zz1 CaTRmg;RtIekaNmaRt . 2 K> edayeRbIlTæplxagelIcUrTajrktémøR)akdén cos 12 nig sin 12 %^> eKmancMnYnkMupøic z1  1  i nig z 2  2 3  2i . 2 k> cUrsresr Z  z 1 .z 2 CaTRmg;BICKNit .

.

x> cUrsresr z 1 , z 2 nig Z  z 1 .z 2 CaTRmg;RtIekaNmaRt . K> edayeRbIlTæplxagelIcUrTajrktémøR)akdén cos 512 nig sin 512 . %&> eK[ z  1  i . cUrsresr z nig z 2007 CaTRmg;RtIekaNmaRt . %*> eK[ z   3  i . cUrsresr z nig z 2007 CaTRmg;RtIekaNmaRt . %(> eK[ z  12  i 23 . cUrsresr z nig z 2007 CaTRmg;RtIekaNmaRt . ^0> eK[cMnYnkMupøic Z  2  3  i k-cUrepÞógpÞat;fam:UDúl | Z | 6  2 . x-bgðajfa Z  2(1 cos6  i.sin6) rYcTajrkTRmg;RtIekaNmaRtén Z . K-Tajbgðajfa cos 12 

eroberogeday lwm plÁún

6 2 4

. - TMBr½140 -

sikSaKNitviTüaedayxøÜnÉg

cMnYnkMupøic

^!> eK[cMnYnkMupøic Z  2  1  i k-cUrepÞógpÞat;fam:UDúl | Z | 2 . 2  2 . x-bgðajfa Z  2(1  cos 4  i. sin 4 ) rYcTajrkTRmg;RtIekaNmaRtén Z . K-Tajbgðajfa cos 8  2 2 2 . ^@> eKmancMnYnkMupøic ³ 3i ni g Z 2  ( 3  1)  i( 3  1) . Z1  2 k> cUrsresr U  Z1 .Z 2 CaragBICKNit . x> cUrsresr U nig Z1 CaragRtIekaNmaRt rYcTajrkm:UDúl nigGaKuym:g; éncMnYnkMupøic Z 2 . K>cUrTajbgðajfa cos 12  6 4 2 nig sin 12  6 4 2 .

^#> eK[cMnYnkMupøic z  1  2i 3 .

cUrrkm:UDúl nigGaKuym:g;éncMnYnkMupøic U 

eroberogeday lwm plÁún

z 2012 1 z

2

.

- TMBr½141 -

sikSaKNitviTüaedayxøÜnÉg

^$> eK[cMnYnkMupøic Z n 

cMnYnkMupøic

n 2  n  1  i( 2n  1) (n 2  n  1) 2  ( 2n  1) 2

Edl n CacMnYnKt;FmμCati . k> bgðajfa Z n  n 1 i  n  11  i . x> KNna S n  Z 0  Z1  Z 2  .....  Z n edaysresrlTæpl Edl)anCaragBICKNit . ^%> eKmansIVúténcMnYnkMupøic (Z n ) kMnt;eday ³ 1 i 3 1 i 3 3i 3 ni g Edl n  IN . Z0  Z n 1  Zn  2 2 2 k> eKtag U n  Z n  1 cMeBaHRKb; n  IN . cUrbgðajfa U n1  1  2i 3 .U n , n  IN x> cUrsresr U n CaTRmg;RtIekaNmaRt . K> cUrRsaybBa¢ak;fa ³ Z n  2 cos

(n  1)  (n  1) (n  1)  cos i . sin   6 6 6 

X> cUrkMnt;TRmg;RtIekaNmaRtén Z n . ^^> ebI z CacMnYnkMupøicEdlepÞógpÞat;TMnak;TMng ³ z 2n  (1  z ) n cMeBaHRKb; n  IN enaHcUrRsaybBa¢ak; facMnYnkMupøic z nig 1  1z manm:UDúlesμIKña . eroberogeday lwm plÁún

- TMBr½142 -

sikSaKNitviTüaedayxøÜnÉg

cMnYnkMupøic

^&> edaHRsaysmIkar z 3  z 3  i | z |2  1  i . ^*> eKmancMnYnkMupøic   1  2i 7 nig   1  2i 7 k> sresrsmIkardWeRkTIBIrEdlman  ,  Cab¤s . x> tag n  Z : S n   n   n . cUrRsayfa S n 2  S n1  2S n  0 ? ^(> eK[cMnYnkMupøic ³

 Z 1  a1  i .b 1   Z 2  a 2  i .b 2  Z  a  i .b  3 3 3

Edl a1 , a 2 , a 3 , b1 , b 2 , b 3   

CacMnYnBit . kñúglMhRbkbedaytMruyGrtUnrm:al; (o, i , j , k )  

 eK[RtIekaN ABC mYyEdl AB (a1 , a 2 , a 3 ) nig AC (b 1 , b 2 , b 3 ) . k>cUrkMnt;RbePTénRtIekaN ABC kalNaeKmanTMnak;TMng Z 12  Z 22  Z 23  0 . x> cUrRsayfaebI ABC CaRtIekaNEkgsm)aTkMBUl A

Z 16  Z 62  Z 63 enaHeK)an (Z1 .Z 2 .Z 3 )  3 2

eroberogeday lwm plÁún

.

- TMBr½143 -

sikSaKNitviTüaedayxøÜnÉg

&0> eK[cMnYnBit x Edl x  2  k , k  Z k> cUrRsaybBa¢ak;fa ³ 1  3 tan 2 x  i .( 3 tan x  tan 3 x ) 

x> eRbITMnak;TMngxagelIcUrTajbgðajfa ³ tan 3x 

cMnYnkMupøic

cos 3x  i sin 3x cos 3 x

3 tan x  tan 3 x

1  3 tan 2 x K> cUrsresr tan( 3  x) nig tan( 3  x) CaGnuKmn_én tan x . 3x X> Taj[)anfa tan( 3  x) tan( 3  x)  tan . tan x n g>cUrKNnaplKuN Pn   [tan( 3  3n a). tan( 3  3n a)] k 0

.

&!> edaHRsaysmIkar | z | i.z  1  3i Edl z CacMnYnkMupøic . &@> edaHRsaysmIkar log 5 (z.z )  z  3 (1  2i ) . &#> edaHRsaysmIkar | z  1  i |  iz  22  4i . &$> eK[sIVúténcMnYnkMupøic (Z n ) kMnt;eday ³ 1 Z 0  1 nig Z n 1  ( Z n  | Z n | ) Edl n  IN 2 KNna Z n edaysresrlTæpleRkamTRmg;RtIekaNmaRt .

eroberogeday lwm plÁún

- TMBr½144 -

sikSaKNitviTüaedayxøÜnÉg

cMnYnkMupøic

   i sin 9 CaTRmg;RtIekaNmaRt . &%> sresrcMnYnkMupøic z  49 4 cos  i sin 9 9   i cos  &^> sresrcMnYnkMupøic z  11  sin CaTRmg;RtIekaNmaRt . sin   i cos  &&> enAkñúgbøg;kMupøic (xoy) tamcMNuc M' (z' ) CarUbPaBén M(z  4  4i ) tambMElgvilp©it O nigmMu   6 . cUrkMMNt; z' ? cos

n n 2 2 (cos

n ) . 4 4 n n n n &(> bgðajfa ( 3  i )  2 (cos 6  i. sin 6 ) . *0> eK[cMnYnkMupøic z  cos 6  i. sin 6 k> edAcMnuc z , z 2 , z 3 , z 4 , z5 , z 6 , z7 elIrgVg;RtIekaNmaRt .

&*> bgðajfa (1  i )n 

 i sin

x> kMNt;témø 1  z  z 2  z 3  z 4  ....  z11 . *!> eKmancMnYnkMupøic 3  2i nig 4  5i EdlmanrUbPaBtagedaycMNucerogKña A nig B . k> KNnaRbEvg AB x> rkkUGredaencMNuckNþal AB .

eroberogeday lwm plÁún

- TMBr½145 -

sikSaKNitviTüaedayxøÜnÉg

cMnYnkMupøic

*@> cMNuc A , B nig C CarUbPaBerogKñaéncMnYnkMupøic 2  i ,  1 nig 3  2i . R)ab;RbePTénRtIekaN ABC . *#> cUrrk nig sg;sMNMucMNuc M manGahVik z EdlepÞógpÞat;lkçxNÐxageRkam ³ k> | z  1  i | 2 x> | 2z  3  2i | 4 K> | z  i | 2 X> zz  1i  1 g> zz  1i  2 c> ziz21i  1 *$>k-cMeBaHRKb;cMnYnkMupøic z1 nig z 2 cUrbgðajfaeKmansmPaB ³ 2 ( | z1 |2  | z 2 |2 ) | z1  z 2 |2  | z1  z 2 |2

x-cUrbkRsaysmPaBenHtamEbbFrNImaRt . *%> cUrkMNt;cMnYnkMupøic z edaydwgfa | z |  | 1z |  | 1  z | . *^> cUredaHRsaysmIkarkñúgsMNMukMpøic z6  *&> eK[   e  1 

2



i

4 7

1 i 3i

.

. cUrRsaybBa¢ak;fa ³

2 1 

4



3 1 

6

 2

**> edaHRsaykñúgsMNMucMnYnkMupøic z6  8i rYcTajrktémø cos 12 nig sin 12 *(> eK[ z  x  i.y Edl x , y  IR . bgðajfa | 2z |  | x |  | y | . eroberogeday lwm plÁún

- TMBr½146 -

sikSaKNitviTüaedayxøÜnÉg

cMnYnkMupøic

Éksareyag !> esovePAKNitviTüafñak;TI 11 kMritx esovePAKNitviTüafñak;TI 12 rbs;RksYgGb;rMyuvCn nigkILa ¬e)aHBum Mathématiques Géométrie (Terminales C et E) ( Genevieve HAYE , Bernard RANDÉ , Eric SERRA)

$> Complex Numbers from A to…Z ( Titu Andreescu , Dorin Andrica )

eroberogeday lwm plÁún

- TMBr½147 -

complex-numbers.pdf

There was a problem previewing this document. Retrying... Download. Connect more apps... Try one of the apps below to open or edit this item.

1MB Sizes 11 Downloads 229 Views

Recommend Documents

No documents